Math Basic Training

You might also like

Download as pdf or txt
Download as pdf or txt
You are on page 1of 50

CONVERTING REPEATING DECIMALS TO FRACTIONS EUCLIDEAN ALGORITHM

Consider the decimal string of the form: EUCLEDIAN ALGORITHM – a method for solving the
greatest common divisor (gcd) of two numbers.
0.d 1 d 2 d 3 K d n r1 r2 r3 K rm
PROCESS:

Where d1d 2 d 3 K d n is a finite-stringed decimal


a) Given two numbers A and B, divide the larger
followed by an infinitely-repeated decimal string, number by the smaller number and find the
r1 r2 r3 K rm . remainder ( r1 ).
b) Next, divide the smaller number by the
We can convert this to an equivalent fraction by this obtained remainder ( r1 ) and again, get the
simple rule: remainder ( r2 ).
c) Continue this process until the latest
0.d1 d 2 d 3 K d n r1 r2 r3 K rm remainder ( rn ) exactly divides the previous
(d1d 2 d 3 K d n r1r2 r3 K rm ) − (d1d 2 d 3 K d n ) remainder rn −1 (latest divisor). This number will
=
999... 000... serve as the gcd of A and B.
m − digits n − digits

EXAMPLE:
EXAMPLES:
* What is the gcd of 231 and 315 ?
* What is the equivalent fraction for the repeating
decimal 0.65141414…? Solution:
1 2 1 1
Solution: 231 315 84 231 63 84 21 63
6514 − 65 − 63 − 63
0.65141414K = 0.6514 = − 231 − 168
9900 21 0
84 63
6449
0.65141414K =
9900
Therefore, gcd(231,315) = 21 .
* What ratio of integers is represented by 0.076923 ?
* What is the gcd of 71162 and 64515 ?
Solution:
76923 − 0 76923 Solution:
0.076923 = = 1 1 1
999999 999999
64515 71162 6647 64515 4692 6647
1
0.076923 = − 64515 − 59823 − 4692
13
6647 4692 1955
2 2 2
PRACTICE EXERCISES: 1955 4692 782 1955 391 782
− 3910 − 1564 − 782
Express the following into their simplest fraction
representation: 782 391 0

1. 0.14344 Therefore, gcd (64515,71162) = 391 .

2. 0.142857 PRACTICE EXERCISES:

Find the following:


3. 3.14152

1. gcd (650,702)

2. gcd(5577,12155)

1
3. Let: * The roots of x 3 − 11x 2 − x + 2 = 0 are r, s and t. What
m = 1111K111 (2004 1' s ) is the value of r 2 st + rs 2 t + rst 2 ?
and
n = 1111K111 (666 1' s ) Solution:
Find gcd (m, n ) . For the given cubic:
−11
sum of roots = r + s + t = − = 11
th 1
n DEGREE GENERAL POLYNOMIAL EQUATIONS
product of roots = rst = (− 1)3 = −2
2
For a general nth degree general polynomial 1
equation in x; n ∈ Ζ + : ( ) Thus:
r 2 st + rs 2 t + rst 2 = rst (r + s + t )

a n x n + a n −1 x n −1 + a n − 2 x n − 2 + K + a 2 x 2 + a1 x + a 0 = 0 r 2 st + rs 2 t + rst 2 = (− 2 )(11)
with roots r1 , r2 , r3 , K , rn −1 , rn . r 2 st + rs 2 t + rst 2 = −22

We say that:
PRACTICE EXERCISES:
i. Sum of the roots (taken one at a time):
a n −1 1. Find the product of the solutions to the equation:
r1 + r2 + r3 + K + rn −1 + rn = −
an 2 x 3 + x 2 − 13 x + 6 = 0 ?
ii. Sum of all possible product of roots taken 2 at a
2. If a and b are the roots of the polynomial
time:
a equation x 2 + 2 = 0 , then what is the value of
r1 r2 + r1 r3 + r1 r4 + K + rn − 2 rn + rn −1 rn = n − 2
an a5 + b5 ?
iii. Sum of all possible product of roots taken 3 at a
time: 3. If the roots of x 3 + ax 2 + bx + c = 0 are three
a n−3 consecutive positive integers, then what are all
r1r2 r3 + r1r2 r4 + r1r2 r5 + K + rn−3 rn−1rn + rn−2 rn−1rn = −
an a2
possible values of ?
…and so on until… b +1

iv. Product of the roots (taken n at a time):

r1 ⋅ r2 ⋅ r3 ⋅ K ⋅ rn −1 ⋅ rn = (− 1)n
a0
an

EXAMPLE:

* If 5 is the sum of the roots of 3 x 2 + bx + 9 = 0 , what is


the sum of the roots of 9 x 2 + bx + 3 = 0 ?

Solution:
For 3 x 2 + bx + 9 = 0 :
b
sum of roots = 5 = −
3
b = −15

Thus, for 9 x 2 + bx + 3 = 0 :
b −15
sum of roots = − =−
9 9
5
sum of roots =
3

2
TEST 1: f (x ) = x 4 − 4 x 3 + 18 x 2 − 28 x − 51

1. Express the sum of the repeating decimals: Find the product of the other three zeros.

0.686868K + 0.777777 K
9. If m and n are the roots of x 2 + mx + n = 0 ,
m ≠ 0 ; n ≠ 0 , then m + n is what?
as a repeating decimal.

10. The polynomial:


2. Let:

F = 0.48181K x 2 − 9x + 3 = 0

with digits 8 and 1 repeating. Then, when F is Has roots r and s. If x 2 + bx + c has roots
written as a fraction in lowest terms, the 2 2
r and s , then solve for b and c.
denominator exceeds the numerator by how
much?
11. If a, b and c are the roots of:
3. Express:
x3 − x −1 = 0
0.420352035K
1 1 1
Compute: + + .
as a fraction in lowest terms. 1− a 1− b 1− c

4. A and B are digits such that the repeating 12. Let a, b, c, d and e be the roots of the quintic
decimal: equation:

0. ABABAB K =
8 x 5 − 6 x 4 + 5 x 3 − 10 x 2 + 11x − 7 = 0
A + 2B
Find the exact value of:
Find A and B.
(a + 1)(b + 1)(c + 1)(d + 1)(e + 1)
5. The equation:

13. Find the sum of the roots of the polynomial:


2x − 1 − x − 1 = 1
2001
1 
has two solutions. The sum of these solutions is x 2001 +  − x  =0
2 
what?

6. If m and n are the roots of the equation: 14. There are three pairs of real numbers
(x1 , y1 ), (x 2 , y 2 ) and (x3 , y 3 ) that satisfies both

ax 2 + bx + c = 0 x 3 − 3xy 2 = 2005 and y 3 − 3x 2 y = 2004 .

1 1 Compute:
Find + .
m n
 x  x  x 
1 − 1 1 − 2 1 − 3 
7. Find the sum of all values of x which satisfy the  y1  y 2  y3 
following system:

15. If a, b and c be the roots of :


y = x 2 − 5x + 4
x 3 − 9 x 2 + 11x − 1 = 0
z = x − 12 x + 35
2
and suppose that s = a + b + c . Find the
yz =1
value of s − 18s − 8s .
4 2

8. The complex number 1 − 4i is a zero of:

3
LINEAR INEQUALITIES IN TWO VARIABLES QUADRATIC INEQUALITIES

 If A, B and C are real numbers with A and B not  A quadratic inequality is an inequality of the form:
both zero, then:
Ax 2 + Bx + C > 0
Ax + By ≤ C
where a, b and c are real numbers with A ≠ 0 .
is called a linear inequality in two variables. In The inequality symbols < , ≤ , and ≥ may also be
place of ≤ , we can also use ≥ , < , or > .
used.

Graphing Linear Inequalities in Two Variables: Strategy for Solving a Quadratic Inequality with a Sign
Graph:
 Solve the inequality for y, then graph y = mx + b .
1. Write the inequality with zero on the right.
CASE 1: y > mx + b is satisfied above the line. 2. Factor the quadratic polynomial on the left.
CASE 2: y = mx + b is satisfied on the line itself. 3. Make a sign graph showing where each
factor is positive, negative, or zero.
CASE 3: y < mx + b is satisfied below the line. 4. Use the rules for multiplying signed numbers to
determine which intervals satisfy the original
SAMPLE PROBLEMS: inequality.
5. Write the solution set using interval notation or
the like.
Graph each inequalities:
SAMPLE PROBLEMS:
1. 3x − 4 y > 7 .
1 Find the solution interval of the inequalities below:
2. y > x − 3 and y<− x+2
2
3. 2 x − 3 y ≤ −6 or x + 2y ≥ 4 . 1. 2x 2 + 5x ≤ 3 .

4. y > x + 1 and y<x−2. Solution:


2 x 2 + 5x − 3 ≤ 0
 A compound inequality is two inequalities joined
by a connective “and” or the connective “or”
(2 x − 1)(x + 3) ≤ 0
where:
Using Sign Graph:
“and” means INTERSECTION of the two solution
sets of the compound inequality. For two-variable  1 1 
inequalities, the solution set must satisfy BOTH (− ∞, − 3] − 3 , 2  2 ,∞
   
inequalities.
(2 x − 1)(x + 3) (+ ) (− ) (+ )
“or” means UNION of the two solution sets of the
compound inequality. For two-variable
 1
inequalities, the solution set must satisfy either one Thus, the solution interval is − 3 ,  .
or both of the inequalities. 2 
2. x + 6x + 9 > 0 .
2

SAMPLE PROBLEMS:
Solution:
Graph the compound inequalities below:
(x + 3)2 > 0
1 Since the square of every nonzero real
1. y > x − 3 and y<− x+2. number is greater than zero, there is only one
2
2. 2 x − 3 y ≤ −6 or x + 2 y ≥ 4 . number that fails to satisfy this inequality and that
number is the solution to x + 3 = 0 . So the solution
3. y > x + 1 and y < x − 2 .
set is all real numbers except -3.

Thus, the solution interval is (− ∞,−3) ∪ (− 3, ∞ ) .

4
Thus the solution interval
3. x − 10 x + 25 ≥ 0 .
2
( ) (
is − ∞ , 2 − 10 ∪ 2 + 10 , ∞ . )
Solution:
7. x 2 + 5x + 8 > 0 .

( x − 5)2 ≥ 0 . Solution:
Since the square of every real number is The discriminant is −7 . So the equation has no
greater than or equal to zero, all real numbers real solutions and x 2 + 5 x + 8 does not change in
satisfy the inequality.
sign. So x 2 + 5 x + 8 > 0 is either correct for all real

Thus, the solution interval is (− ∞, ∞ ) . numbers or incorrect for all real numbers. Select a
test point say 0, to get 0 2 + 5(0 ) + 8 > 0 which is
4. 4 x 2 − 20 x + 25 < 0 .
true. So the inequality is satisfied by 0 and all other
Solution: real numbers.

(2 x − 5)2 < 0 . Thus, the solution interval is (− ∞, ∞ ) .


Since no real number has a negative square,
there is no solution to this inequality.
8. − x 2 + 3x − 5 ≥ 0 .
Thus, the solution set is the empty set φ .
Solution:
The discriminant is found to be -11. So the
5. 9x 2 − 6x + 1 ≤ 0 . quadratic equation has no real solution. Select a
test point, say 0, to get − 0 2 + 3(0) − 5 ≥ 0 , which is
Solution:
false. So no real numbers satisfy the inequality.
(3x − 1)2 ≤ 0 .
Since no real number has a negative square, Thus, the solution set is the empty set φ .
to (3 x − 1) < 0 .
2
there are no solutions But
RATIONAL INEQUALITIES
(3x − 1)2 = 0 does have one solution and that is 1 .
3
 The inequalities:
1 
Thus the solution set is   .
3 x+2 2x − 3 2 1
≤ 2, ≤ 0 and ≥
6. x 2 − 4x − 6 > 0 . x−3 x+5 x + 4 x +1

Solution: are examples of rational inequalities. When we


The quadratic polynomial is prime but we can solve equations that involve rational expressions,
solve x 2 − 4 x − 6 = 0 by the quadratic formula: we usually multiply each side by the LCD.
However, if we multiply each side of an inequality

4 ± 16 − 4(1)(− 6 )
by a negative number, we must reverse the
x= = 2 ± 10 inequality, and when we multiply by a positive
2(1) number, we do not reverse the inequality. For this
reason we generally do not multiply inequalities
Using Sign Graph: by expression involving variables.

x 2 − 4x − 6 Strategy for Solving a Rational Inequality with a Sign


Graph:
(− ∞ , 2 − 10 ) (+ )
1. Rewrite the inequality with zero on the right-
( 2 − 10 , 2 + 10 ) (− )
2.
hand side.
Use only addition and subtraction to get an

(2 + 10 , ∞) (+ ) 3.
equivalent inequality.
Factor the numerator and denominator if
possible.
4. Make a sign graph showing where each
factor is positive, negative, or zero.

5
5. Use the rules for multiplying and dividing x−2
(− ) (+ ) (− ) (+ )
signed numbers to determine the intervals (x + 1)(x + 4)
that satisfy the original inequality.
6. Write the solution set using interval notation or Thus the solution interval is (− 4 , − 1) ∪ [2 , ∞ ) .
the like.

SAMPLE PROBLEMS:
ABSOLUTE VALUE EQUATIONS
Find the solution set:
BASIC ABSOLUTE VALUE EQUATIONS
Absolute Value Equivalent
x+2 Solution Set
1. ≤2. Equations Equation
x−3
x = k (k > 0) x = k or x = −k {k ,−k}
Solution: x =0 x=0 {0}
We do not multiply each side by x − 3 .
Instead, subtract 2 from each side to get 0 on the x = k (k < 0 ) none φ
right:

x+2 SAMPLE PROBLEMS:


−2≤0
x−3
x + 2 2x − 6 Solve each equation:
− ≤0
x−3 x−3
−x + 8 1. 3x − 5 = 7 .
≤0
x−3
Solution:
Using Sign Graph: First rewrite 3 x − 5 = 7 without absolute value:
3x − 5 = 7 or 3x − 5 = −7
(− ∞ , 3) (3 , 8] [8 , ∞ ) x=4 or x = −2 3
−x + 8
(− ) (+ ) (− )
x−3  2 
The solution set is − , 4 .
 3 
Thus the solution interval is (− ∞ , 3) ∪ [8 , ∞) .
2. 2(x − 6) + 7 = 0 .
2 1
2. ≥ .
x + 4 x +1 Solution:
Since 0 is the only number whose absolute
Solution: value is 0, the expression within the absolute value
We do not multiply by the LCD as we do in bars must be 0.
1
solving equations. Instead, subtract from
x +1 2(x − 6 ) + 7 = 0
each side: 5
x=
2
2 1
− ≥0
x + 4 x +1 5 
x−2 The solution set is  .
≥0 2 
(x + 1)(x + 4)
3. − 5 3x − 7 + 4 = 14 .
Using Sign Graph:

Solution:

(− ∞ , − 4) (− 4 , −`1) (− 1, 2] [2 , ∞ ) First subtract 4 from each side to isolate the


absolute value expression:

6
− 5 3x − 7 + 4 = 14
 7
− 5 3x − 7 = 10 Thus, the solution interval is  − ∞ , −  ∪ (− 1, ∞) .
 3
3x − 7 = −2
3. 5 − 3x ≤ 6 .

Thus, there is no solution because no quantity


Solution:
has a negative absolute value.
−6 ≤ 5 − 3x ≤ 6
−11 ≤ −3x ≤ 1
4. 2x − 1 = x + 3 .
11 1
≥x≥−
3 3
Solution:
1 11
Two quantities have the same absolute value − ≤x≤
3 3
only if they are equal or opposites. So we can
write an equivalent compound equation:
 1 11 
Thus, the solution interval is − , .
2x − 1 = x + 3 or 2 x − 1 = − ( x + 3)  3 3
x=4 or x = −2 3
4. 3 + 7 − 2x ≥ 3 .
 2 
The solution set is − , 4 .
 3  Solution:
Subtract 3 from each side to isolate the
absolute value expression:
ABSOLUTE VALUE INEQUALITIES

7 − 2x ≥ 0
BASIC ABSOLUTE VALUE INEQUALITIES (k>0)
Absolute Value Equivalent
Solution Set Because the absolute value of any real number is
Inequality Inequality
greater than or equal to 0, the solution set is the
x >k −k > x > k (− ∞,−k ) ∪ (k , ∞ ) set of all real numbers.
x ≥k −k ≥ x ≥ k (− ∞, − k ] ∪ [k , − ∞ )
Thus, the solution interval is (− ∞ , ∞ ) .
x <k −k < x < k (− k , k )
x ≤k −k ≤ x ≤ k [− k, k ] 5. 5 x − 12 < −2 .

SAMPLE PROBLEMS: Solution:


We write an equivalent inequality only when the
Find the solution interval of each inequality below: value of k is positive. With -2 on the right-hand
side, we do not write an equivalent inequality.
1. x −9 < 2. Since the absolute value of any quantity is greater
than or equal to 0, no value for x can make this
Solution: absolute value less than -2. The solution set is an
−2 < x − 9 < 2 empty set.
7 < x < 11
SECOND-DEGREE INEQUALITIES
Thus, the solution interval is (7 , 11) .
 A second-degree inequality is an inequality
involving squares of at least one of the variables.
2. 3x + 5 > 2 . Second-degree inequalities are graphed in the
same manner as linear inequalities.
Solution:
SAMPLE PROBLEMS:
−2 > 3 x + 5 > 2
−7 > 3 x > −3
Graph the second-degree inequalities below:
7
− > x > −1
3

7
1. y < x 2 + 2x − 3 .
2. x2 + y2 < 9 .
x2 y2
3. − >1.
4 9

SYSTEMS OF SECOND-DEGREE INEQUALITIES

 System of second-degree inequalities follow the


“and” rule. So the resulting graph will be the
intersection of the graphs.

SAMPLE PROBLEM:

Graph the system of inequalities:

y2 x2 x2 y2
− >1 ; + <1
4 9 9 16

LOGARITHM

Logarithm - is the power or exponent to which the


base must be raised in order to produce the
number.

Types of Logarithm:

1. Briggsian Logarithm – a type of logarithm named


after Henry Briggs, a 17th century British
mathematician. It is more commonly known
as “common logarithm”. Sometimes it is
called as “decadic logarithm” because this is
a logarithm in base 10.

2. Naperian Logarithm – more commonly known as


“natural logarithm” where the base is the
number e. Named after the Scottish
mathematician John Napier of Merchistoun.

Some Properties of Logarithm:

1. log b ( xy ) = log b x + log b y


x
2. log b   = log b x − log b y
 y
3. log b a n = n log b a
4. M = b log b M = e ln (M )
log c a
5. log b a =
log c b
6. log b (1) = 0 ; ln (1) = 0
7. log (10) = 1 ; ln (e ) = 1

8
TEST 2:
10. x1 and x 2 are the roots of the equation:
1. What is the largest integer k such that: x −10

100 x −10 − 110 x log ( x ) = −1000


3 2 1 2 k 1
⋅ ⋅ ⋅ ⋅K⋅ ≥ ?
2 1 2 3 k +1 8 Find the value of x1 + x 2 .

2. Suppose x satisfies: 11. Let a and b be two positive integers such that b is
a multiple of a. If:
x 2 − 2x − 3 = x 2 − 2x + 5
b 9a
 b 2  a
log  + log  =1
Find all possible real values for x.
a  b
 
3. Solve for x:
then, b 2 − a 2 equals what?
log 3 x − 2 < log 3 (4 − x ) .
12. x, y and z are real numbers greater than 1 and w
is a positive real number. If:
4. In the xy-plane, the solution set to:
log x w = 24 ; log y w = 40 ; log xyz w = 12
24 10 120
> +
y x xy
Find log z w .

splits up naturally into four regions, only one of


which is bounded. Find the area of that bounded 13. If:
region. log 2 (log 8 x ) = log 8 (log 2 x )

5. Let x and y be real numbers. Find the smallest Find (log 2 x ) .


2
possible value of:

2 − x + x − y + y − 2004 14. Find in terms of n:

1 1 1


log n   ⋅ log n −1   ⋅ K ⋅ log 2  
6. If x 2 + 2 x + n > 10 for all real numbers x, then what  
2  
3 n
are all possible real values for n?

15. Given that:

7. What is the sum of all integers x that satisfy:


( )
log b a 2 = 3
x
−5≤ ≤ 10 .
π ( )
What is the value of log a b 2 ?

8. What is the value of the sum:

 3  4  99   100
log(2) + log  + log  + K+ log  + log 
 2  3  98   99 

9. If:
log(2 ) = b ; log(3) = c ; log(7 ) = a

Find the value of:


45
log100 7
16

9
SEQUENCE AND SERIES
Sn =
(143 − 35 + 1)(143 + 35) = (109)(178)
2(1) 2
 Sequence – also known as progression. These are S n = 9701
ordered sets of quantities that are in one-to-one
correspondence with the set of positive integers 1
* In an arithmetic sequence, the first term is and
and are governed by a certain condition. 10
1
 Series – is an expression which is an indicated sum the hundredth term is . Find the sum of the first 50
5
of the terms of a given sequence.
terms.

ARITHMETIC SEQUENCE
Solution:
First, solve for the common difference (d):
a100 = a1 + (100 − 1) d
 A sequence in which the difference between
each term is constant wherein that constant is
called the “common difference” (d). 1 1
= + 99d
5 10
IMPORTANT PARAMETERS: 1
d=
990
a. nth term (a n ) :
Thus the sum of the first 50 terms is:
a n = a1 + (n − 1) d
50   1   1 
S 50 = 2  + (50 − 1) 
2   10   990 
b. Sum of the first n terms (S n ) :
1235
i. Given a1 and a n : S 50 =
198
Sn =
n
(a1 + a n )
2
HARMONIC SEQUENCE
ii. Given a1 and d :
n
Sn = [2a1 + (n − 1) d ]  A sequence whose reciprocals of the terms form
2 an arithmetic sequence.
iii. Given a1 , a n and d :
(a n − a1 + d )(a n + a1 ) IMPORTANT PARAMETER:
Sn =
2d
Harmonic Mean – is the reciprocal of the average of
the reciprocals of two or more numbers. Let us
c. r th term from n arithmetic means from a to b: denote it as H.
a(n − r + 1) + br
r th term =
n +1 Given numbers: a1 , a 2 , a 3 , K , a n
d. Sum of n arithmetic means between a and b:

sum =
n
(a + b ) n(a1 a 2 a 3 ⋅ K ⋅ a n )
2 H=
a1 a 2 a 3 K a n −1 + a1 a 2 a 3 K a n − 2 a n + a 2 a 3 K a n −1 a n
Arithmetic Mean – most commonly known as
“average”. It is the average of n given numbers. Let us GEOMETRIC SEQUENCE
denote it as A.
 A sequence in which any term after the first term
Given numbers: a1 , a 2 , a 3 , K , a n is the product of the preceding term and a
constant number. The constant number is called
the “common ratio” usually denoted by r.
a1 + a 2 + a 3 + K + a n
A=
n IMPORTANT PARAMETERS:

SAMPLE PROBLEMS: a. nth term (g n ) :


( )
g n = g 1 r n −1
Sum of the first n terms (S n ) :
* What is the sum of all integers from 35 to 143
b.
inclusive?
Solution: i. Given g1 and r :

10
g1 1 − r n
Sn =
( ) ; where r ≠ 1 sum =
6720 + 3744 10
1− r 96
ii. Given g1 , g n and r : sum = 70 + 39 10
g − rg n
Sn = 1
1− r
iii. For infinite geometric series (n = ∞ ) :
g1
S∞ =
1− r
c. m th term from n geometric means from a to b:
n +1
m th term = a n − m +1b m
d. Sum of n geometric means between a and b:
ab − a n +1 a n b
sum =
n +1
a nb − a
e. Product of n geometric means between a and b:
product = (ab )n

Geometric Mean – also known as “mean proportion”.


It is the nth root of the product of n numbers. Let
us denote it as G.

Given numbers: a1 , a 2 , a 3 , K , a n

G = n a1 a 2 a 3 K a n

SAMPLE PROBLEMS:

1 1 1
* Find the sum of the sequence: 1 + + + +K
3 9 27

Solution:
1
This is an infinite geometric series with r = . Thus:
3
1
S∞ =
1
1−
3
2
S∞ =
3

* What is the sum of 5 geometric means between 8


and 125?

Solution:

125(8) − 86 8 5 (125) 1000 − 8 2 5 (5)


sum = =
6
8 5 (125) − 8 2 5 (5) − 8

1000 − 8 2 5 (5) 2 5 (5) + 8


sum = ⋅
2 5 (5) − 8 2 5 (5) + 8

11
TEST 3:

1. What is the sum of all even numbers from 2 to 10. Evaluate:


2008 inclusive?
1 1 1 1 1 1 1 1 1
1+ + − + + − + + − +K
2. Take a look at the sequences 2 4 8 16 32 64 128 256 512

3, 20, 37, 54, 71, …


and 11. Two towns are 50 miles apart. They travel towards
16, 27, 38, 49, 60, 71, … each other at rates 30 mph and 20 mph
respectively. A bumblebee flying at the rate of 50
Each has 71 as the common term. Determine the mph starts out just as the faster train departs the
next common number after 71. train station and flies to the slower train. The bee
then turns around and goes back to meet the
faster train. Then it turns around again, etc and
3. Find the sum:
keeps flying back and forth between the trains
meet. How far does the tired bumblebee fly?
1 − 4 + 9 − 16 + K + 99 2 − 100 2
12. Evaluate:
4. Alice, Bob and Charlie go to the park on January
1, 2008. Alice returns on January 7, 13, … , every (2 + 1)(2 2 + 1)(2 4 + 1)(28 + 1)(216 + 1) + 1
sixth day. Bob returns every fourth day and Charlie
returns every fifteenth day. When is the first day 2 30
after January 1, 2008 when all three go to the
park? 13. If five geometric means are inserted between 8
and 5832, the fourth term in the geometric series is
5. The roots of the polynomial what?

P(x ) = 36 x 3 + 36 x 2 + 11x + 1 14. Evaluate:

forms an arithmetic progression. Find the


( ) ( ) ( ) ( )
2 3 −1 − 4 3 −2 + 6 3 −3 − 8 3 −4 + K
difference of the smallest root from the biggest
one. 15. Find the mean proportion of 4 and 36.

16. Compute:
6. An ant moves 1 cm rightward, then 2 cm
downward, then 4 cm leftward, then 7 cm
upward, then 11 cm rightward and so on and so
forth. In its 2008th movement, find the distance 2 2 3 2 4 2 5 2 ⋅K
and direction.
17. The third term of a geometric progression is 36
7. The numbers 1, a, 9 forms an arithmetic sequence. and the fifth term is 16. Find the tenth term.
The numbers 1, b, 9 forms a geometric sequence.
Determine the value of a+b.
18. Evaluate:
8. Find the 7th term in the sequence:
1 1 1 1 1 1 1 1 1
1+ + + + + + + + + +K
1 1 1 2 3 4 6 8 9 12 16 18
, , ,K
2 6 10
19. a, b and c are positive integers forming an
increasing geometric sequence, b − a is a square
9. A rubber ball is made to fall from a height of 50
and log 6 a + log 6 b + log 6 c = 6 . Find a + b + c .
2
feet and is observed to rebound of the
3
20. Evaluate:
distance it falls. How far will the ball travel before
coming to rest if the ball continues to fall in this
1 1 1 1
manner?
 1  3  1  9  1  27  1  81
  ⋅   ⋅   ⋅   ⋅K
 2 4 8  16 

12
FIBONACCI SEQUENCE BINET’S FORMULA

Fibonacci Sequence – a sequence of integers starting  Binet’s Formula, formulated by the French
from 1 wherein the next term is equal to the sum mathematician Jacques Philippe Marie Binet was
of the two terms immediately preceding it. used to determine the nth Fibonacci number by a
closed form formula.
“0, 1, 1, 2, 3, 5, 8, 13, 21, 34, 55, 89, …”
Binet’s Formula is mathematically stated as:

Some Properties of Fibonacci Sequence:


an − bn
Fn =
1. If Fn is the nth Fibonacci number, then: a−b
Fn = Fn −1 + Fn − 2
where a and b are the two roots of the quadratic
2. For every nth Fibonacci number, the greatest
number that can divide them is the nth Fibonacci equation x 2 − x − 1 = 0 . So that:
number ( Fn ).
n n
Fn 1+ 5   
3. As n becomes larger, the ratio tend to   − 1− 5 
Fn −1  2   2 
   
reach the “Golden Ratio”. Fn =
5
1+ 5
Golden Ratio = ≈ 1.61803K
2
LUCAS SEQUENCE
4. No two consecutive Fibonacci numbers have a
factor greater than 1.
gcd (Fn −1 , Fn ) = 1 Lucas sequence - is a particular generalization of the
Fibonacci numbers and Lucas numbers. Lucas
5. For any prime p ≠ 2 , 5 , it is known that either
sequences are named after French
F p −1 or F p +1 is divisible by p. mathematician Edouard Lucas.
6. The greatest common divisor of two Fibonacci
numbers is also a Fibonacci number. Lucas numbers are an integer sequence named after
gcd (Fm , Fn ) = Fd ; where d = gcd (m, n ) the mathematician François Édouard Anatole
Lucas (1842–1891), who studied both that
sequence and the closely related Fibonacci
Formulas regarding Fibonacci numbers:
numbers (both are Lucas sequences). Like the
Fibonacci numbers, each Lucas number is
1. Sum of the first n Fibonacci numbers ( S n ): defined to be the sum of its two immediate
S n = F1 + F2 + F3 + K + Fn = Fn + 2 − 1 previous terms, i.e. it is a Fibonacci integer
sequence. Consequently, the ratio between two
2. Sum of the first n Fibonacci number with odd consecutive Lucas numbers converges to the
indices: golden ratio.
F1 + F3 + F5 + K + F2 n −1 = F2 n
3. Sum of the first n Fibonacci numbers with even  However, the first two Lucas numbers are L0 = 2
indices: and L1 = 1 instead of 0 and 1, and the properties
F2 + F4 + F6 + K + F2 n = F2 n +1 − 1 of Lucas numbers are therefore somewhat
4. Alternating Series of Fibonacci numbers: different from those of Fibonacci numbers.

F1 − F2 + F3 − F4 + K + (− 1)n +1 Fn = 1 − (− 1)n Fn −1
The sequence of Lucas numbers begins:
5. Square of a Fibonacci number: “2, 1, 3, 4, 7, 11, 18, 29, 47, 76, 123, ...”
Fn 2 = Fn −1 Fn +1 − (− 1)n ;n≥2
6. Sum of squares of two consecutive Fibonacci The nth Lucas number ( Ln ) is expressed in closed form
numbers: formula as:
Fn 2 + Fn +1 2 = F2 n +1
n n
7. Sum of the squares of the first n Fibonacci 1+ 5   
numbers: Ln = a n + b n =   + 1− 5 
 2   2 
F1 2 + F2 2 + F3 2 + K + Fn 2 = Fn Fn +1    

13
RELATIONSHIP BETWEEN FIBONACCI 25 75025 167761
AND LUCAS NUMBERS
So the 25th Fibonacci number is 75025.
The Lucas numbers are related to the Fibonacci
numbers by the identities: SUM OF THE COEFFICIENTS OF A POLYNOMIAL

Fn + Ln For a general polynomial P ( x ) :


1. Fn +1 =
2
5 Fn + Ln P(x ) = a n x n + a n−1 x n−1 + K + a 2 x 2 + a1 x + a 0
2. Ln +1 =
2
3. F2 n = Fn Ln where a n , a n −1 , K , a 2 , a1 , a 0 are the coefficients of
4. L2 n = Ln 2 − 2(− 1)n the polynomial, we can say that the sum of these
coefficients is equal to:
Ln
Take Note: As n approaches infinity, the ratio
Fn sum of coefficients = P(1)

approaches 5 .
For a polynomial in more than one
variable P( x, y, z , K)
SAMPLE PROBLEMS:

 What is the 14th Lucas number? sum of coefficients = P(1, 1, 1, K)

Solution: SAMPLE PROBLEM:

To find the 14th Lucas number, we will use the  What is the sum of the coefficients in the
expansion of the polynomial (2 x − 1)
identities above. We will consider the values for k 100
?
in the table below as k = 1, 2, 3, 6, 7, 14 .

Solution:
k Fk Lk
1 1 1 Using the definition above:
2 1 3 sum of coefficients = P(1) = [2(1) − 1]100
3 2 4
sum of coefficients = [2(1) − 1]100 = 1
6 8 18
7 13 29
14 377 843  What is the sum of the coefficients in the
expansion of the polynomial (2 x − 3 y + 2 z )
143
?
So the 14th Lucas number is 843.
Solution:
 What is the 25th Fibonacci number?
Again, using the definition above:
sum of coefficients = P(1, 1, 1, K) = [2(1) − 3(1) + 2(1)]143
Solution:

sum of coefficients = 1
Again, using the identities above, we will
consider values for k as k = 1, 2, 3, 6, 12, 24, 25 .
NESTED FORM OF A POLYNOMIAL

k Fk Lk
Sometimes, when evaluating a polynomial in
1 1 1 one variable to a certain value, we encounter the
2 1 3 difficulties in expanding each term because the
3 2 4 exponents were quite large. Thus, it is recommended
6 8 18 to use the nested form of a polynomial.
12 144 322
24 46368 103682 For the polynomial say in x:

14
P(x ) = a n x n + a n −1 x n −1 + K + a 2 x 2 + a1 x + a 0
We can use the advantage of evaluating a
nested polynomial to find the remainder
The equivalent “nested form” is:
mentally…

P(x ) = (((a n x + a n −1 )x + a n − 2 )x + K)x + a 0


The equivalent nested polynomial of the problem
is:
SAMPLE PROBLEMS:
x 5 − 4 x 4 + 2 x 3 − 5 x 2 + 13 x − 23 =
((((x − 4)x + 2)x − 5)x + 13)x − 23
 Transform the cubic polynomial x 3 + 2 x 2 + 3 x + 4
into its equivalent nested form.
Evaluating at x = 3 , we have:
Solution: ((((3 − 4)3 + 2)3 − 5)3 + 13)3 − 23 = −56

x 3 + 2 x 2 + 3x + 4 = ((x + 2)x + 3)x + 4 Thus remainder = P(3) = −56 .

 Evaluate the quintic polynomial :


x 5 − 4 x 4 + 2 x 3 − 5 x 2 + 13x − 23 at x = 3 .

Solution:

The equivalent nested polynomial of the problem


is:
x 5 − 4 x 4 + 2 x 3 − 5 x 2 + 13 x − 23 =
((((x − 4)x + 2)x − 5)x + 13)x − 23

Evaluating at x = 3 , we have:
((((3 − 4)3 + 2)3 − 5)3 + 13)3 − 23 = −56

The answer is -56.

REMAINDER THEOREM

Remainder Theorem gives us the ease of


determining the remainder when a polynomial is
divided by a linear divisor.

Thus, when the polynomial:

P(x ) = a n x n + a n −1 x n −1 + K + a 2 x 2 + a1 x + a 0

is divided by a certain linear polynomial (x-a), then we


can simply substitute x = a on the polynomial so that:

remainder = P(a )

SAMPLE PROBLEMS:

 What is the remainder when the polynomial


x 5 − 4 x 4 + 2 x 3 − 5 x 2 + 13x − 23 is divided by
x −3?

Solution:

15
TEST 4:

1. Find the numerical coefficient of the term (a + bi )5 = 5 + 4i


involving b 4 in the expansion of a 2 − 2b ( )
10
.
12. Compute:
2. What is the constant term when you expand:
log3−5i (− 4 + 7i )
10
 2 2 
x − 
 y 3  13. Expand:
 
2008
 
3. Evaluate: − 2 + 2 i
 2 2 
 
10  10  10  10  10 
  +   +   + K +   +  
0 1 2  9  10  14. Evaluate:

4. Compute: i
i
 2007 1  2007 1  2007 1  2007 1  2007
  −   +   − K +   −   15. Evaluate:
 0  2  1  3 2  2007 2006 2008 2007

i 0! + i 1! + i 2! + i 3! + K + i 2008!
5. How many odd numbers are there in the
150th row of the arithmetic triangle?
16. Given that:
6. How many terms are there in the expansion
of: 1+ 5 −1+ 5
cos 36 o = ; cos 72 o =
4 4
(x 2
+ y2 )
2007

Simplify:

( )
2008
7. Simplify:  10 + 2 5 + − 1 + 5 i 
 
(1 + i )100
17. Compute:

8. Evaluate:
ii
i + i 2 + i 3 + i 4 + K + i 2008200720 06

18. How many even numbers are there in the


9. What is: 129th row of the arithmetic triangle?

log(−1)

( )( )
10. If 4cis 210 o 3cis120 o is expressed in
rectangular form a + bi , what is the value of
a
the ratio ?
b

11. Find all possible ordered pairs (a, b ) in the


equation:

16
COUNTING TECHNIQUES choice of whether to include pickles, mustard,
ketchup, onions, tomatoes, lettuce, or cheese.
How many different hamburgers are available at
a. Fundamental Counting Principle
Windy’s?

“If event A has m different outcomes and event B


has n different outcomes, then there are mn
Solution:
different ways for events A and B to occur.”
There are 3 outcomes to the event of choosing
the amount of meat. For each of the condiments,
b. Permutations
there are 2 outcomes: whether or not to include it.
A Permutation is any ordering or arrangement of
So the number of different hamburgers is:
distinct objects in a linear manner.

# of hamburgers = 3 ⋅ 2 ⋅ 2 ⋅ 2 ⋅ 2 ⋅ 2 ⋅ 2 ⋅ 2
i. Permutations of n Things n at a Time:
n Pn = n! # of hamburgers = 384
ii. Permutations of n Things r at a Time:
n!  How many different license plates are possible if
Pr =
n
(n − r )! each plate consists of 2 letters followed by a 4-
digit number? Assume repetitions in the letters or
numbers are allowed and any of the 10 digits may
c. Combinations be used in each plate of the 4-digit number.
A Combination is an un-ordered collection of
distinct elements, usually of a prescribed size and Solution:
taken from a given set. Since there are 26 choices for each of the 2 letters
and 10 choices for each of the numbers, by the
i. Combinations of n Things r at a Time: Fundamental Counting Principle, the number of
 n n! license plates is:
n Cr =   =
r
  (n − r )!⋅r!
ii. Combinations with Repetition: # of plates = 26 ⋅ 26 ⋅ 10 ⋅ 10 ⋅ 10 ⋅ 10
 n + r − 1 (n + r − 1)! # of plates = 6760000
n + r −1 C r =   =
 r  r! (n − 1)!
 Eight prize winners will be randomly selected from
25 people attending a sales meeting. There will be
For example, if you have ten types of donuts
a first, second, third, fourth, fifth, sixth, seventh and
(n) on a menu to choose from and you want
eighth prize, each prize being of lesser value than
three donuts (k) there are the one before it. In how many different ways can
(10 + 3 − 1)! / 3!(10 − 1)! = 220 ways to choose. the prizes be awarded, assuming no one gets
more than one prize?
d. Labeling Problem
In a labeling problem, we count the number of Solution:
ways to put labels on distinct objects. If there are The number of ways in which these prizes can be
n distinct objects that are to be given a label with awarded is precisely the number of permutations
each object getting exactly one label and there of 25 things taken 8 at a time,
are r1 labels of the first type, r2 labels of the
25! 25!
second type, … , and rk labels of the kth type,
25 P8 = =
where
(25 − 8)! 17!
r1 + r2 + r3 + K + rk = n 25 P8 = 43609104000
Then, the number of ways to assign the labels to
the objects is:  In how many ways can a committee of 4 people
be chosen from a group of 12?
n!
r1!r2 !r3!K rk ! Solution:
Choosing 4 people from a group of 12 is the same
SAMPLE PROBLEM: as choosing a subset of size 4 from a set of 12
elements. So the number of ways to choose the
 At Windy’s Hamburger Palace you can get a committee is the number of combinations of 12
single, double, or triple burger. You also have a things taken 4 at a time:

17
a = 2mn ; b = m 2 − n 2 ; c = m2 + n2
12! 12! m > n , m and n are positive integers
12 C 4 = = where
(12 − 4)!⋅4! 8!⋅4!
12 C 4 = 495 But in the case of primitive Pythagorean triples, it
was Leonardo of Pisa (commonly called Fibonacci)
who device some conditions fro obtaining these
 What is the coefficient of a 4b 2 in the binomial
triples.
expansion of (a + b ) ?
6

Some Characteristics of Primitive Pythagorean Triples:


Solution:

coefficien t = 6 C 4 =
6!
= 15 a. If (a, b, c ) is a primitive Pythagorean triple, then
4!⋅2! one of the integers a and b is even and the other
is odd.
 How many different arrangements are there for b. In a primitive Pythagorean triple (a, b, c ) , either a
the 11 letters in the word MISSISSIPPI? or b is divisible by 3.

Solution: Now, for the method on solving Primitive Pythagorean


This problem is a labeling problem if we think of Triples:
the 11 positions for the letters as 11 distinct objects
to be labeled. There are 1 M-label, 4 S-labels, 4 I- Based on the argument of Fibonacci, we will use the
labels and 2 P-labels. So the number of ways to following formulas:
arrange the letters in MISSISSIPPI is:

a = 2st ; b = s 2 − t 2 ; c = s2 + t 2
11!
# of arrangements = = 34650
1! 4!4!2!
wherein for each value of s ≥ 2 , we have taken those
values of t that are relatively prime to s, less than s and
 What is the coefficient of a 3b 2c in the expansion “even” whenever s is odd.
of (a + b + c ) ?
6

SAMPLE PROBLEM:
Solution:
This is also an example of a labeling problem:  Find all primitive Pythagorean triples wherein 15 is
one of the leg.
6!
coefficient = = 60
3!2!1! Solution:
It is obvious that a ≠ 15 because a by nature is
PRIMITIVE PYTHAGOREAN TRIPLES even. Thus:
b = s 2 − t 2 = 15
A Pythagorean triple (a,b,c) is said to be
(s + t )(s − t ) = 15 where 15 = 5(3) = 15(1)
primitive if the three integers a, b and c are pair-wise
relatively prime, that is each pair has gcd of 1. Suppose that:
s + t = 15 and s − t = 1 thus s = 8 and t = 7
From ancient times, there are some methods for s + t = 5 and s − t = 3 thus s = 4 and t = 1
finding Pythagorean triples (a,b,c). Some of these are: So that the primitive Pythagorean triples are:
(a, b, c ) = (8,15,17 )
a. Pythagoras’ Method:
(a, b, c ) = (112,15,113)
a = 2 n + 1 ; b = 2n 2 + 2n ; c = 2n 2 + 2n + 1
for n>0. FINDING ALL PYTHAGOREAN TRIPLES

b. Plato’s Method: This method was devised by Brahmagupta for finding


a = 2n ; b = n 2 − 1 ; n 2 + 1 all Pythagorean triples involving a given measure of
for n >1. one leg (a).

c. Euclid and Diophantus’ Method:

18
For a triplet of the form: reminiscent of ordinary equality. According to
a +b =c
2 2 2 Carl Friedrich Gauss: “If a number n measures the
difference between two numbers a and b, then a
where a and b are the legs of a right triangle and c is
and b are said to be congruent with respect to n;
the hypotenuse,
if not, incongruent”

We can look for all possible combinations of b and c


Test for Congruence:
for a given value of a.

Let n be a fixed positive integer. Two integers a and b


a. Look for all values of m that are factors of a 2 , and are said to be congruent modulo n symbolized by:
where (a − m) is a positive even number. Also, it
must satisfy the following: a ≡ b(mod n )
a2
i. If a is even, must be even.
m a. If n divides the difference a − b , that is, provided
a 2 that a − b = kn for some integer k, or;
ii. If a is odd, must be odd. b. If a and b have the same non-negative
m
remainder when divided by n.
b. For each value of m, calculate the value of b.
Thai is:
Elementary Properties of Congruence:
a 2 − m2
b=
2m
Let n > 0 be fixed and a, b, c and d be arbitrary
c. Similarly, calculate the corresponding value of c:
integers. Then the following properties holds:
c=b+ m
d. In addition, you can actually determine the radius
of the inscribed circle based on the values of a, b a. a ≡ a(mod n ) .
and c. That is: b. If a ≡ b(mod n) , then b ≡ a(mod n) .

inradius =
a+b−c c. If a ≡ b(mod n) and b ≡ c(mod n) , then a ≡ c(mod n) .
2 d. If a ≡ b(mod n) and c ≡ d (mod n) , then
a + c ≡ b + d (mod n) and ac ≡ bd(mod n) .
e. If a ≡ b(mod n) , then a + c ≡ b + c(mod n) and
SAMPLE PROBLEM:

ac ≡ bc(mod n) .
 Find all possible Pythagorean triples produced for
a given leg whose measure is 15 units. f. If a ≡ b(mod n) , then a k ≡ bk (mod n) for any positive
integer k.
Solution:  n
If ca ≡ cb(mod n) and gcd(c, n) = d , then a ≡ b mod  .
a = 15 = 3(5) thus a 2 = 225 = 32 ⋅ 52
g.
 d
Possible values for m are:
m = 1, 3, 5 and 9
BASIC APPROACH IN FINDING REMAINDERS
a2 − m2
a m b= c=b+ m
2m
If a1 ⋅ a2 ⋅ a3 ⋅ K ⋅ an is divided by b provided that a1 ,
15 1 112 113
15 3 36 39 a2 , a3 , … , an are positive integers, then:
15 5 20 25
15 9 8 17 remainder = [a1 (mod b ) ⋅ a2 (mod b ) ⋅ a3 (mod b ) ⋅ K
⋅ an (mod b )](mod b )
Thus, possible triplets (a, b, c ) are (15,112,113) ,
(15,36,39) , (15,20,25) and (15,8,17) . SAMPLE PROBLEM:

CONGRUENCE  What is the remainder when the product of 31, 41


and 47 is divided by 11?
Congruence – can be viewed as a generalized form
of equality, in the sense that its behavior with Solution:
respect to addition and multiplication is

19
remainder = [31(mod 11) ⋅ 41(mod 11) ⋅ 47(mod 11)]mod11 where p1 , p2 , p3 ,K, pk are distinct primes and
remainder = (9 ⋅ 8 ⋅ 3)(mod 11) a1 , a 2 , a3 , K , a k are positive integers. So that
remainder = [72 (mod 11) ⋅ 3](mod 11)
remainder = (6 ⋅ 3)(mod 11) = 18 (mod 11) τ (n ) = (a1 + 1)(a2 + 1)(a3 + 1)L (ak + 1)
remainder = 7
b. SIGMA ( σ ) FUNCTION
 What is the remainder when 8103 is divided by 13? This function determines the sum of all possible
positive integer divisors of n with prime
Solution: factorization defined above.
remainder = 8103 (mod 13)
( )
remainder = 6451 ⋅ 8 (mod 13) σ (n ) =
p1a1 +1 − 1 p2 a 2 +1 − 1 p3 a3 +1 − 1
⋅ ⋅ ⋅
remainder = (12 ⋅ 8)(mod 13)
51 p1 − 1 p2 − 1 p3 − 1

remainder = (144 ⋅ 12 ⋅ 8)(mod 13)


25
K⋅
pk a k +1 − 1

remainder = (144 ⋅ 96)(mod 13)


25 pk − 1

remainder = (1 ⋅ 5)(mod 13)


25
c. EULER’S PHI (φ ) FUNCTION
remainder = 5 This function determines the number of positive
integers less than the integer n that is relatively
 Since today is Sunday, August 17,2008, what day prime to n, that is, their greatest common divisor is
of the week will it be 2 2897 days from now? 1.

Solution:
 1  1   1 
Since there are 7 days in a week, we only just φ (n ) = n ⋅ 1 − 1 −  ⋅ K ⋅ 1 − 
 p1  p2  
 p 
k 
have to find the remainder when 2 2897 is divided
by 7. So:
SAMPLE PROBLEM:
remainder = 2 2897
(mod 7 )
remainder = 8 ( 965
⋅ 2 2 )(mod 7 )
 How many positive integer divisors do 125 have?

remainder = 4 Solution:
The prime factorization of 125 is:

( )
Therefore 4 days after a Sunday is a THURSDAY. 5
125 = 2 2 ⋅ 3 = 210 ⋅ 35
TAU, SIGMA AND PHI FUNCTIONS So that the number of positive integer divisors of
125 is given by the τ function.
In number problems, particularly those that involve
huge numbers along with their prime factorizations,
( )
τ 125 = (10 + 1)(5 + 1) = 66
 What is the sum of all positive integer divisors of
we are often encountered with questions such as
“how many positive integer factors…”, “what is the 125 ?
sum of the factors of…?” and “how many positive
Solution:
integers less than a number that does is relatively
The answer is given by the sigma function:
prime to that number?” The three functions below will
( ) 22 −−11 ⋅ 33 −−11 = 2047 ⋅ 364
11 6
solve these problems!
σ 12 5 =

a. TAU (τ ) FUNCTION σ (12 ) = 745108


5

This function determines the number of positive


integer divisors of a given integer n with prime  How many positive integers less than 12 5 that are
factorization: relatively prime to this number?

n = p1a1 ⋅ p2 a 2 ⋅ p3 a 3 ⋅ K ⋅ pk a k Solution:
Using Euler’s Phi Function, we get:

20
( )  1  1
φ 12 5 = 12 5 ⋅ 1 − 1 −  = 12 5 ⋅ ⋅
1 2 ( )
remainder = 31 + 171 (mod 20)
 2  3 2 3 remainder = 20(mod 20)
φ (12 ) = 82944
5
remainder = 0

EULER’S THEOREM

Euler’s Theorem is a method of finding remainders by


congruence using the “Euler’s Phi Function” denoted
by φ (n ) .

According to Fermat’s Little Theorem:

“If p is a prime, a is a positive integer and gcd ( p, a ) = 1 ,


then:
a p −1 ≡ 1(mod p ) ”

Note that since p is prime, we can rewrite this


congruence as:

aφ ( p ) ≡ 1(mod p ) , because φ ( p ) = p − 1

But we can actually generalize this to a result where


the modulus is not necessarily a prime. This
generalization is known as the “Euler’s Theorem”.

EULER’S THEOREM:

If m is a positive integer and a is an integer with


gcd (a, m ) = 1 , then:
a φ (m ) ≡ 1(mod m )
In addition, if a and b are positive integers and
gcd(a, b ) = 1 then:
aφ (b ) + bφ (a ) ≡ 1(mod ab )

SAMPLE PROBLEM:

 What will be the remainder when 2317 + 17 25 is


divided by 20?
Solution:
(
remainder = 2317 + 17 25 (mod 20) )
remainder = (3 17
+ 17 25
)(mod 20)
Using Euler’s Theorem:
 1  1
φ (20) = 20 ⋅ 1 − 1 −  = 8
 2  5
So that:
17(mod 8) ≡ 1 ; 25(mod 8) ≡ 1
Therefore:

21
TEST 5:
where i = −1 .
1. Given the digits 0, 2, 3, 5, 6, 7, 8, 9. How many 5-
digit numbers will be formed if the resulting 14. What is the remainder when:
numbers are even, less than 70000 and without
repetition of digits?
2005 2006 + 2006 2007 + 2007 2008
2. On a multiple-choice test of 10 questions, each
question has 5 choices. A student is certain of the is divided by 25?
answers to 4 questions but is totally baffled by the
other 6 questions. What is the probability that the 15. How many positive integer divisors do 5050 have?
student will get a score of 5 or more on the test?
16. Find the sum of all possible positive integer divisors
3. In how many ways can you arrange the letters of of 216.
the word MATHEMATICS?
17. How many positive integers less than 243 are
4. What is the coefficient of a 4b 3c 2 d in the relatively prime to 243?
expansion of (a + b + c + 2d ) ?
10

18. What is the remainder when:


5. in how many ways can you arrange the words in
this sentence? 111333 + 333111

6. From the equation below: is divided by 7?

x 2 + 12 2 = z 2 19. Evaluate:

Find all positive ordered pairs of integers ( x, z )


2008
 π π
 cos + i sin 
such that gcd (x, z ) = 1 .  6 6

7. How many Pythagorean triplets can you form if 20. Find the 6th roots of 1.
the measure of one leg is equal to 36?
21. Compute
8. Find all ordered pairs (a, b ) satisfying the
equation: log(− 1)

a 2 + 900 = b 2 22. Evaluate:

where a and b are positive integers. ii

9. Find all primitive Pythagorean triples wherein one 23. Evaluate:


of the leg is 18?
i
i
10. What is the radius of the inscribed circle in a
triangle whose side lengths are 9 cm, 40 cm, and
41cm.

11. Find the remainder when the product of 143, 206,


198 and 77 is divided by 17.

12. My birthday is June 23, 1982 and that is a


Wednesday. On what day will it be 13 2008 days
from that date?

13. Evaluate:

2007 2 2008
i2 + i 2007 + i 2007

22
TRIGONOMETRIC FUNCTIONS

Trigonometry - (from Greek Τριγωνοµετρία "tri = three"


+ "gon = angle" + "metr[y] = to measure") is a
branch of mathematics that deals with triangles,
particularly those plane triangles in which one
angle has 90 degrees (right triangles).
Trigonometry deals with relationships between the
sides and the angles of triangles and with the
trigonometric functions, which describe those
relationships.

ANGLE CONVERSIONS wherein a point P( x, y ) along the perimeter


of the unit circle corresponds to the
360 o = 2π radians = 6400 mils = 400 grads value P(cos A, sin A) .

THE SIX TRIGONOMETRIC FUNCTIONS TRIGONOMETRIC IDENTITIES

Here are some of the identities that you must


know and understand when solving problems in
trigonometry:

Pythagorean Identities:

sin 2 A + cos 2 A = 1
opposite a
1. sin A = = sec 2 A − tan 2 A = 1
hypotenuse c
csc 2 A − cot 2 A = 1
adjacent b
2. cos A = =
hypotenuse c Complement Identities:
opposite sin A a
3. tan A = = =
adjacent cos A b π 
sin − A  = cos A
hypotenuse 1 c 2 
4. csc A = = =
opposite sin A a π 
tan  − A  = cot A
hypotenuse 1 c 2 
5. sec A = = =
adjacent cos A b
π 
adjacent 1 b sec − A  = csc A
6. cot A = = =  2 
opposite tan A a
Odd and Even Functions:
Students often use mnemonics to remember facts
and relationships in trigonometry. For example, the sin (− A) = − sin A ; csc(− A) = − csc A
sine, cosine, and tangent ratios in a right triangle can
be remembered by representing them as strings of cos(− A) = cos A ; sec(− A) = sec A
letters, as in SOH-CAH-TOA. tan (− A) = − tan A ; cot (− A) = − cot A
Sine = Opposite ÷ Hypotenuse
Cosine = Adjacent ÷ Hypotenuse
Tangent = Opposite ÷ Adjacent Sine, Cosine and Tangent of a Sum:

SPECIAL ANGLE VALUES ON A UNIT CIRCLE sin ( A ± B ) = sin A cos B ± cos A sin B
cos( A ± B ) = cos A cos B m sin A sin B
The figure below tells us of all the special tan A ± tan B
angles along a unit circle (a circle of radius 1) and tan ( A ± B ) =
1 m tan A tan B
their respective positions on the Cartesian plane.
Half-Angle Identities:

23
SAMPLE PROBLEMS:
A 1 − cos A
sin =±  Evaluate: sin 75 + sin 15 .
2 2
A 1 + cos A Solution:

( ) ( )
cos
2 2 sin 75 o + sin 15 o = sin 45 o + 30 o + sin 45 o − 30 o

A
sin
A
2 = sin A = 1 − cos A
(
= sin 45 o cos 30 o + cos 45 o sin 30 o + )
tan =
2
cos
A 1 + cos A sin A (sin 45 o
cos 30 − cos 45
o o
sin 30 )
o

2  2  3   2  1 
=  +   +
 2  2   2  2 
Double-Angle Identities:
 2  3   2  1 
sin 2 A = 2 sin A cos A  −  
 2  2   2  2 
cos 2 A = cos 2 A − sin 2 A
6
= 2 cos 2 A − 1 sin 75 o + sin 15 o =
2
= 1 − 2 sin 2 A
2 tan A sin 4 x − sin 3 x
tan 2 A =  Express in terms of sin x and cos x .
1 − tan A2 cos 4 x + cos 3 x

Solution:
Sum to Product Identities:
Using sum to product identities on the
numerator and denominator of the given expression,
 A± B  Am B
sin A ± sin B = 2 sin   cos  we have:
 2   2 
 A+ B   A−B  4 x − 3x   4 x + 3x 
cos A + cos B = 2 cos  cos  2 sin  cos 
 2   2  sin 4 x − sin 3 x  2   2 
=
 A+ B   A− B  cos 4 x + cos 3 x  4 x + 3x   4 x − 3x 
cos A − cos B = −2 sin   sin   2 cos  cos 
 2   2   2   2 
x
sin
Product to Sum Identities: sin 4 x − sin 3 x 2 = tan x
=
cos 4 x + cos 3 x x 2
cos
1
sin A cos B = [sin ( A + B ) + sin (A − B )] 2
2 sin 4 x − sin 3x sin x 1 − cos x
= =
cos 4 x + cos 3x 1 + cos x
cos A sin B = [sin ( A + B ) − sin ( A − B )]
1 sin x
2
If 3 cos x = 4 sin x , find the value of 900 cos x sin x .
cos A cos B = [cos( A + B ) + cos( A − B )]
1 
2
Solution:
sin A sin B = [cos( A − B ) − cos( A + B )]
1
2 3 cos x = 4 sin x
sin x 3 3 4
= ; sin x = ; cos x =
Stereographic (or parametric) Identities: cos x 4 h h
sin 2 x + cos 2 x = 1
2T
sin A =  3
2
 4
2
1+T 2   +   =1 ⇒ h = 5
 
h h
1−T 2
cos A =
1+ T 2 Therefore:
A  4  3 
where T = tan 900 cos x sin x = 900   = 432
2  5  5 

24
INVERSE TRIGONOMETRIC FUNCTIONS Therefore:
3 π π
tan A = ; − < A<
Here are the table of the six inverse 3 2 2
trigonometric function and their parameters:
sin A 3 3 3
= ; sin A = ; cos A =
cos A 3 h h
Range of sin 2 A + cos 2 A = 1 ⇒ h = 2 3
Usual Domain
usual Thus:
Name notatio Definition of x for
principal
n real result 1 3
value sin A = ; cos A =
2 2
An angle satisfying this is:
y=
−π/2 ≤ y ≤ 3 π
arcsine arcsin( x = sin(y) −1 to +1 A = Arc tan =
π/2 3 6
x)

  8  4 
y=  Evaluate: tan  Arc cos  + Arc sin   .
arccosine arccos x = cos(y) −1 to +1 0≤y≤π   
17  5 
(x) Solution:
Let:
 8   4 
y= x = tan  Arc cos  + Arc sin 
−π/2 < y <  17   5 
arctangent arctan x = tan(y) all 
π/2
(x) and;
8 4
A = Arc cos ; B = Arc sin
y= 17 5
arccotang
arccot x = cot(y) all 0<y<π Therefore:
ent
(x) 8 4
cos A = ; sin B =
17 5
y= 0 ≤ y < π/2 It follows that by familiarity of the Pythagorean triples:
−∞ to −1 15 3
arcsecant arcsec x = sec(y) or π/2 < y sin A = ; cos B =
or 1 to ∞
(x) ≤π 17 5
So that:
15 4
arccoseca
y=
−∞ to −1
−π/2 ≤ y < tan A = ; tan B =
arccsc x = csc(y) 0 or 0 < y ≤ 8 3
nt or 1 to ∞
(x) π/2 Going back to the problem:
  8   4 
x = tan  Arc cos  + Arc sin   = tan ( A + B )
Note: When we take the inverse trigonometric   
17  5 
function of a number, this means that we are 15 4
finding its equivalent angle along the unit circle. +
tan A + tan B 8 3
If the inverse trigonometric function requires x= =
1 − tan A tan B  15  4 
principal values, then we must follow the 1 −   
restrictions on the table above.  8  3 
77
x=−
SAMPLE PROBLEMS: 36

3 TRIANGLE SOLUTIONS
 Find Arc tan .
3
Solution:  When dealing with Right Triangles, just use
Since Arc denotes principal value, then: Pythagorean Theorem or the six trigonometric
Functions (sin, cos, tan, etc.).
3
A = Arc tan
3  When the triangle is oblique, you can these
following Laws:

25
LAW OF SINES:
a b c
= = = 2R
sin A sin B sin C

where: R is the radius of the circumcircle.

LAW OF COSINES:
a 2 = b 2 + c 2 − 2bc cos A
b 2 = a 2 + c 2 − 2ac cos B
c 2 = a 2 + b 2 − 2ab cos C

LAW OF TANGENTS:
1 
tan  ( A + B )
a+b 2 
=
a−b 1 
tan  ( A − B )
2 

SUMMATION OF SINE AND COSINE

  nx   (n + 1)x  
n sin   ⋅ sin  
  2   2 

i =1
sin (ix ) =
x
sin  
2

  nx   (n + 1)x  
n sin  ⋅ cos  
  2   2 
∑i =1
cos(ix ) =
 x
sin  
2

36 DEGREES AS A SPECIAL ANGLE

Here are some useful facts regarding the


o
trigonometric function of 36 .

1+ 5
cos 36 o =
4

−1+ 5
cos 72 o =
4

26
TEST 6:
12. Evaluate:
1. Simplify the expression:

cos A + cos B sin A + sin B   15   7 


+ cos  Arc tan   − Arc tan 
sin A − sin B cos A − cos B  8  25 

2. Evaluate: 13. Given that:

sin 2 15 o + sin 2 75 o P = A sin t + B cos t


Q = A cos t − B sin t
3. A pole cast a shadow of 15 meters long when the
angle of elevation of the sun is 45 o . If the pole has Find: P 2 + Q 2 .
leaned 15 o from the vertical directly toward the
sun, what is the length of the pole? 14. Evaluate:

4. Find the value of:


1 1
2 Arc tan  + Arc tan 
 3 7
sin 0 o + sin 1o + sin 2 o + K + sin 89 o + sin 90 o
cos 0 o + cos 1o + cos 2 o + K + cos 89 o + cos 90 o 15. If A, B and C are angles of a triangle and
tan A + tan B + tan C = 5.67 . Find the value of
5. Find all possible values of x: (tan A)(tan B )(tan C ) .
sin x + sin 2 x + K + sin 10 x = cos x + cos 2 x + K + cos 10 x

6. Find the exact value of :

cos 11.25 o

7. Simplify:

cos{Arc tan[sin ( Arc cot x )]}

8. Solve for all values of x:

 2x + 1   2x − 1  π
Arc tan  + Arc tan 
  3 = 4

 3   

9. Evaluate:

Arc sin x + Arc cos x

10. Find the value of P:

P = sin 2 1o + sin 2 2 o + sin 2 3 o + K + sin 2 90 o

11. If:

x tan 39 o
x + tan 219 o + tan 115 o =
cot (−65 o )

Then what is the value of Arc tan x ?

27
CHINESE REMAINDER THEOREM 21 y 2 ≡ 1(mod 5)
→ b2 = 1
y 2 ≡ 1(mod 5)
- This technique is used to determine the least positive

15 y 3 ≡ 1(mod 7 )
number that satisfies a given set of congruences.
→ b3 = 1
y 3 ≡ 1(mod 7 )
( )
If gcd mi , m j = 1 for i = j , then the system:

x ≡ a1 (mod m1 )
n

x ≡ a 2 (mod m 2 )
c. xo = ∑ a M b (mod m)
i =1
i i i

M x o = [(2 ⋅ 35 ⋅ 2) + (3 ⋅ 21⋅1) + (2 ⋅15 ⋅1)](mod 105)


x ≡ a n (mod m n ) x o = 233(mod 105)

is solvable and the solution is unique modulo


x o = 23(mod 105)
m = m1 m 2 m3 ⋅ K ⋅ m n
Thus:
PROCESS: x = 23 + 105k ; k = 0, ± 1, ± 2, K
m
a. Let M i = for i = 1, 2, K , n. The smallest positive x occur when k is zero:
mi
b. M i y ≡ 1 (mod mi ) has solution bi for i = 1, 2, K , n. x = 23 → answer
n
c. xo = ∑a M b
i =1
i i i is the unique solution of the
CALCULATING CALENDAR DATES
system modulo m.
- This method will determine what day of the week
does a certain date fall.
EXAMPLE:

TAKE NOTE: All divisions, except where noted otherwise


 Find the smallest positive integer which leave
are “integer divisions”, in which remainders are
remainderodulis of 2, 3 and 2 when divided by 3, 5 discarded.
and 7 respectively.
STEP 1: Solve for the values of A, Y and M by using the
Solution: formulas:
The integer satisfies the following congruences:
x ≡ 2(mod 3) A=
14 − month
; (month = # of month, 1 for Jan, etc K)
x ≡ 3(mod 5) 12
Y = year − A
x ≡ 2(mod 7 )
M = month + 12 A − 2
Since the moduli m1 = 3 , m 2 = 5 , m 3 = 7 are
pairwise relatively prime, the method of the Chinese STEP 2: Plug the values of y and m into the formula
Remainder Theorem is applicable. below to calculate the day (d).

 31M 
(mod 7 )
m = 3 ⋅ 5 ⋅ 7 = 105 Y Y Y
a. d =  day + Y + − + +
105  4 100 400 12 
M1 = = 35
3 mod 7 means “modulo division”. That is, take the remainder
105 instead of the quotient as your answer. For example,
M2 = = 21
5 ( )
20 mod 7 = 6 because the remainder when 20 is divided
105 by 7 is 6.
M3 = = 15
7
The answer you get for d will correspond to a day of
the week as:
35 y1 ≡ 1(mod 3)
b. 2 y1 ≡ 1(mod 3) → b1 = 2 0 : Sunday
1 : Monday
y1 ≡ 2(mod 3) 2 : Tuesday
3 : Wednesday
4 : Thursday

28
5: Friday  Find the polynomial equation that satisfy the
6: Saturday points (0,−4 ), (1, 1) and (2, 12 ) .

Solution:
EXAMPLE:

Applying Lagrange’s Formula where:


 On what day of the week does June 23, 1982
have fallen?
(x1 , y1 ) = (0,−4)
(x 2 , y 2 ) = (1, 1)
Solution: (x3 , y 3 ) = (2, 12)
STEP 1: June 23, 1982  6/23/1982
y=
(x − 1)(x − 2) (− 4) + (x − 0)(x − 2) (1) + (x − 0)(x − 1) (12)
14 − 6 (0 − 1)(0 − 2) (1 − 0)(1 − 2) (2 − 0)(2 − 1)
A= =0
12
Y = 1982 − 0 = 1982 ( )( ) (
y = −2 x 2 − 3x + 2 − x 2 − 2 x + 6 x 2 − x )
M = 6 + 12(0) − 2 = 4 y = 3x + 2 x − 4
2

STEP 2: DIOPHANTINE EQUATIONS


 1982 1982 1982 31(4) 
d = 23 + 1982 + − + + (mod 7 )
 4 100 400 12  Diophantine Equations – system of equation(s) wherein
d = (23 + 1982 + 495 − 19 + 4 + 10 )(mod 7 )
the number of unknowns is not equal to the
number of available equations.
d = 2495(mod 7 ) = 3
For the Diophantine Equation of the form:
Therefore, June 23, 1982 falls on a WEDNESDAY. ax + by = c
where a, b, c ∈ Ζ and gcd (a, b ) divides c.
LAGRANGE’S INTERPOLATION
PROCESS:
- A general method for finding the explicit polynomial
equation of any degree if we are given set of points a. First, solve for the greatest common divisor of a
that satisfy the polynomial equation. It was and b by Euclidean Algorithm.
formulated by Joseph Louis Lagrange. b. Next, express the result in its Diophantine form.
That is:
ax1 + by1 = gcd(a, b )
The method for each cases are as follows. Take note
of the pattern onto how these equations were NOTE: To easily get this form, reverse the process
constructed. of Euclidean Algorithm.
c. Now, since gcd (a, b ) divides c, multiply gcd (a, b )
a. LINEAR by the necessary multiplier c1 so that:
Given: set of points ( x1 , y1 ) and ( x 2 , y 2 ) c1 (ax1 + by1 ) = c1 gcd(a, b )
x − x2 x − x1 where c1 gcd(a, b ) = c
y= y1 + y2
x1 − x 2 x 2 − x1 d. Lastly, the general solutions for x and y is given by
the equations below:
b. QUADRATIC  b 
Given: set of points ( x1 , y1 ), (x 2 , y 2 ) and ( x 3 , y 3 ) x = c1 x1 +  n
 gcd (a , b ) 
(x − x 2 )(x − x3 ) (x − x1 )(x − x3 )
y= y + y  
(x1 − x 2 )(x1 − x3 ) 1 (x 2 − x1 )(x 2 − x 3 ) 2 y = c1 y1 − 
a
n
 gcd (a , b ) 
(x − x1 )(x − x 2 )
+ y where n is any integer.
(x3 − x1 )(x 3 − x 2 ) 3
… and so on and so forth … EXAMPLE:

EXAMPLE:
 Solve for all values of x and y:
42 x + 70 y = 28

29
Solution:
a. By Euclidean Algorithm:
(
8 3 B + 8 2 O + 8O + K − 5 3 B + 5 2 O + 5O + K )
= 7 B + 7 O + 7O + K
3 2
gcd(42,70) = 14
b. Reversing the Euclidean Algorithm:
44 B −14O = K
42 − 28 = 14
42 − (70 − 42) = 14
and from the original equation we can see that
42(2) + 70(− 1) = 14 0 ≤ B, O, K < 5 .
c. Multiply 2 to both sides of the equation:
42(4) + 70(− 2) = 28 If B = 1 , then O = 3 and K = 2 .
d. Finally, the general solutions are:
 70  In fact, this is the only solution that satisfies the
x = 4 +  n
 14  equation above.
 42 
y = −2 −  n
 14 
for any integer n.

NUMBER BASES

Usually, we write a number a1 a 2 a 3 K a n in its base


ten/decimal representation. That is, if a i is a digit of a
number string, then:

( ) ( ) ( )
a1 a 2 a 3 K a n = a1 10 n −1 + a 2 10 n − 2 + a 3 10 n −3 + K + a n

where the number base is 10.

But sometimes, this is not the case. Actually we can


write the number above in another number base so
that:

(a1 a 2 a 3 K a n )b = a1b n −1 + a 2 b n − 2 + a 3 b n −3 + K + a n

where:
b : number base
a1 , a 2 , a 3 , K , a n < b
a i ≥ 0 ; i = 1, 2, 3, K , n

EXAMPLE:

 Variables B, O and K represent digits in each


number base. If B is not equal to O, O is not equal
to K and K is not equal to B, determine the base
ten sum B + O + O + K such that the values will
satisfy the given equation below (Note: O is the
letter O):
(BOOK )8 − (BOOK )5 = (BOOK )7

Solution:
From their base ten representation:

30
TEST 7:

1. Determine all pairs (a, b ) of real numbers such that 14. A repeating decimal such as x = 3.5474747 K is a
10, a, b, ab is an arithmetic progression. rational number. Convert this into a ratio of two
integers in simplest form.
2. If a is a real number larger than 1, simplify the
expression: 15. If p, q and r are the roots of the equation:
 −log  1   ax 3 + bx 2 + cx + d = 0
 2 −log a a 3  a a  2   1 1 1
  
 Find + + .
  p q r

3. Evaluate: 16. What is the constant term when you expand:


(1 + i )
16
− (1 − i ) 16
 2 2 
x − 5 
7

 x 
4. Five real numbers are chosen and put in order
from smallest to largest. The average of all five is 17. How many odd numbers are there in the 69th row
14. The average of the three middle numbers is of the arithmetic triangle?
only 13. What is the average of the largest and
smallest numbers?
18. What is the sum of the coefficients of the
expansion of (5 x − 4 y + z ) ?
10
5. Solve for x:
4 x −1 = 8 3 x −1
19. Find the term involving x 3 y 2 z when you expand:
6. If a and b are positive real numbers and if (x − y + z )6 .
7 2
a4 = b3 , what is log b a ?
20. Find the 6th roots of 1.
7. Determine the area that is outside the graph of
21. Compute:
x + y ≤ 1 and inside the graph of x 2 + y 2 = 1 . log(1 + i ) .

8. If log(ab ) + log(bc ) + log(ac ) = 10 for positive a, b 22. Evaluate:


and c. What is the value of log(abc ) ? π π
2008

 cos + i sin 
 6 6
9. Jack had an average score of 85 on his first eight
quizzes, and an average score of 81 on his first
nine quizzes. What score did he receive on his 23. When the polynomial p (x ) is divided by x 2 − 1 ,
ninth quiz? the remainder is x + 2 . When p (x ) is divided by

10. How many polynomials are there of the form x 2 − 4 , the remainder is x + 1 . Find the remainder
x 3 − 8 x 2 + cx + d such that c and d are real when p ( x ) is divided by x 2 − 3 x + 2 .
numbers and the three root of the polynomial are
distinct positive integers? 24. When x100 − 1 is divided by x − 2 , the constant
term in the quotient is what?
π
11. If the roots of x − bx + c = 0 are sin
2
and
7 25. Find the sum of all solution(s) x to:
π 2 2x − 1 − x − 5 = 3
cos , then b in terms of c is what?
7
x x 2x
12. If 2 2 + 4 2 = 56 , then what is the value of 2 2 ? 26. How many distinct real solutions are there to the
equation:
13. If log 7 3 = a and log 7 4 = b , find x in terms of a x199 + 2 x198 + 4 x197 + K + 2197 x 2 + 2198 x + 2199 = 0
and b if 9 x = 28 .

31
38. Evaluate:
27. Find the sum of the infinite collection of numbers: 17 17
1 + 5  1 − 5 
1 1 1 1   − 
1 K  2   2 
3 9 27 81    
1 1 1 1
K 5
6 18 54 162
1 1 1
K
39. What is the 2nd largest prime factor of:
( )
36 108 324
1 1
K 38 + 2 8 − 2 6 4
216 648
1
K 40. Find the sum of the finite series:
1296
O 1 1 2 1 2 3  1 2 99 
+  +  +  + +  +K+  + +K 
2 3 3  4 4 4  100 100 100 
28. What is the 25th Fibonacci number?

29. What is the 25th Lucas number?

30. Compute:
143  143 143 143  143
  +   +   + K +   +  
 1   2   3  142  143

31. Evaluate:
2008
∑ n2
n=0

32. If x 2 − 5 x + 1 = 0 , then determine:


x −6 + x 0 + x 6

33. What is the smallest positive integer n > 1 such


that 1 + 2 + 3 + K + n is divisible by 9?

1 1
34. If x+ =8 and xy + = 22 , what is the
y xy
1
numerical value of y + ?
x

35. If a and b are the roots of the equation x 2 + 2 = 0 ,


then a 5 + b 5 equals what?

36. Find all possible real values of x:


(log 3 x )(log 4 x )(log 5 x ) =
(log 3 x )(log 4 x ) + (log 4 x )(log 5 x ) + (log 3 x )(log 5 x )

37. Evaluate:
20 20
1+ 5  1− 5 
  + 
 2   2 
   

32
TEST 8:

1. Find the smallest positive number which 11. Find the smallest counting number that when
leaves a remainder of 5 when divided by 7, a divided by 143, will leave a remainder of 15,
remainder of 8 when divided by 11 and a when divided by 555 will leave a remainder
remainder of 2 when divided by 13. of 357 and when divided by 456 will leave a
remainder of 447.
2. On what day of the week does October 8,
1984 have fallen? 12. How many odd entries are there in the 2009th
row of the Pascal’s triangle?
3. Find the polynomial equation y = f ( x ) of
13. “The 7th day of January in the year 7 A.D. falls
least degree that satisfies the points
(− 1,18), (0,11), (1,6) and (2,3) . on a Friday.”

If not, on what day does it fall?


4. What is the largest negative integer value
that y can take in the equation:
MISCELLANEOUS QUESTIONS:
1001x + 572 y = 286
14. What is the sum of all the digits of all the
if x and y can only take integer values as integers from 1 to 2009?
solution?
15. Evaluate:
5. Explain how is it possible that
143 + 144 = 320 and not 287. sin 2 1o + sin 2 2 o + sin 2 3 o + K + sin 2 359 o + sin 2 360 o

6. You have P10000 to purchase 100 chickens. 16. Find the coordinates of the point on the line
The price of each cock is P600, each hen is 2 x + 3 y = 48 that is nearest to the point (-1, 1).
P300 and each chick worth P10. How many
cocks, hens and chicks can you purchase? 17. A circle is inscribed in a triangle of sides 5, 7
and 6. Find the area of the circle.
7. What day of the week will the Christmas eve
of year 3000 fall? 18. Evaluate:

8. Find the polynomial equation y = f ( x ) of


1 + 2 + 3 +K+ n
least degree that satisfies the points lim
n →∞
3
2
n
(1,0),  0,− 5  and (− 5,5) .
 6
19. Find the length of the arc of the curve
4 y = x 2 from x = 0 to x = 2 .
9. How many pairs (x, y ) of positive integers
satisfy:
20. Evaluate:

2 x + 7 y = 1000
n
i4
lim ∑ n5
n →∞ i =1
10. Assume that b and c are integers greater
2
than 1. In base b, c is written as 10. Then,
2
b when written in base c is what?

33
NUMBER OF ODD ENTRIES IN A ROW OF A PASCAL’S
Pn =
(6 − 2)n2 − (6 − 4)n = 2n 2 − n
TRIANGLE 2
Since, we are looking for the 9th hexagonal number,
In here, we developed the easiest way in then n = 9 . Therefore:
finding the number of odd entries in a particular row
P9 = 2(9)2 − 9
of a Pascal’s Triangle. Here’s how it goes:
P9 = 153
For the nth row of a Pascal’s Triangle:
SUM OF ALL THE DIGITS OF ALL THE INTEGERS FROM
STEP 1: Convert n − 1 in base 2.
1 TO n
STEP 2: Count the number of 1’s in the base 2
representation of n. Denote this as x.
STEP 3: The number of odd entries in this row is found Suppose that we denote S(n) to be the sum of the
by the formula: digits of n where n is the decimal m-stringed number
no. of odd entries = 2 x d m d m −1d m − 2 Kd3d 2 d1 . Therefore the expression:
S (1) + S ( 2) + S (3) + ... + S ( n )
TAKE NOTE: For the nth row of a Pascal’s Triangle, there denotes the sum of all the digits of all the integers
are a total of n entries. from 1 to n.

EXAMPLE: To evaluate this sum with the ease of too many


calculations, I have developed a “FOUR-STEP
 How many odd entries are there in the 2009th row ALGORITHM” to facilitate in faster computations.
of the Pascal’s Triangle?
Since n is the decimal m-string d m d m −1d m −2 Kd3d 2 d1 ,
Solution:
then:

n = 2009 therefore n − 1 = 2008 STEP 1: Find S(n), that is:


STEP 1: In base 2, 2008 = 11111011000
STEP 2: The number of 1’s in 11111011000 is 7. S (n) = d m + d m −1 + d m −2 + K + d3 + d 2 + d1
STEP 3: By applying the formula:
STEP 2: Multiply the leftmost digit of n with the rest of
no. of odd entries = 27 = 128 the digits from its right. Then, multiply the
second leftmost digit with the rest of the
POLYGONAL NUMBERS digits from its right, and so on and so forth
until the second rightmost digit is multiplied
to the rightmost digit of n. Finally, add all the
Polygonal Numbers – these are numbers that can be
resulting products. For example, if n = 14344 :
arranged as a regular polygon. Let us denote it as Pn .
sum of products = 1(4344) + 4(344) + 3(44) + 4(4 )
GENERAL FORMULA:
STEP 3: Consider n except its last digit d1 , that is,
If s is the number of sides in a polygon, the formula for d m d m −1d m −2 Kd4 d3d 2 . Do the operation
the nth s-gonal number is: below:
=
(s − 2)n 2 − (s − 4)n
operation = (d m d m −1d m − 2 K d 4 d3d 2 + d m d m −1d m − 2 K d 4d 3 0
Pn
2
where: + d m d m −1d m − 2 K d 4 00 + d m d m −1d m − 2 K 000 +
s : the number of sides of the polygonal number, s ≥ 3 . K + d m d m −1 0K000 + d m 00K000 )(45)
n: the nth polygonal number, n ≥ 1 .
where “45” is a magic number for this
EXAMPLE: operation.

 What is the 9th hexagonal number? STEP 4: Lastly, add all the positive integers less than
d1 and if the sum exceeds one digit, then
Solution: carry over the exceeding digits onto the next
For the hexagonal number: s = 6 . Substituting to the place value (that is, d 2 ). Same as d1 , add all
given formula above: the positive integers less than d 2 and the
carry over if there is any. If the sum again

34
exceeds one digit, carry it over to the next Amplitude: a
place value. Repeat the same procedure, Phase Shift: d to the left if d < 0 , d to the right if
until you reach d m . For example, n = 143 . d >0.
1 4 3 c. TANGENT : y = c + a tan b(x − d )
1 1 where:
 π 
2 2 Domain :  x x ≠ d + (2n + 1), n ∈ Ζ
 2b 
+ 3
Range: (−∞, ∞ )
6 3 → answer
π
Period:
b
* Finally, add all the results of STEP’s 1 to 4.
π
Vertical Asymptotes: x = d + (2n + 1)
EXAMPLE: 2b
Phase Shift: d to the left if d < 0 , d to the right if
d >0.
 What is the sum of all the digits of all the integers
d. COTANGENT : y = c + a cot b(x − d )
from 1 to 2009?
where:
 nπ 
Solution: Domain :  x x ≠ d + , n ∈ Ζ
 b 
STEP 1: S ( 2009 ) = 2 + 0 + 0 + 9 = 11 . Range: (−∞, ∞ )
π
STEP 2: sum of products = 2(009) + 0(09) + 0(9) = 18 . Period:
b
STEP 3: (200 + 200 + 200 )(45 ) = 27000 . nπ
Vertical Asymptotes: x = d + for n ∈ Ζ
STEP 4: 1001 + (0002 + 0003 + 0004 + K + 0008 ) = 1036 . b
Adding all the results, we have: Phase Shift: d to the left if d < 0 , d to the right if
S (1) + S ( 2) + S (3) + ... + S ( 2009 ) = 11 + 18 + 27000 + 1036 d >0.
S (1) + S (2) + S (3) + ... + S (2009) = 28065 e. SECANT : y = c + a sec b(x − d )
where:
 π 
CIRCULAR FUNCTIONS Domain :  x x ≠ d + (2n + 1), n ∈ Ζ
 2 b 
We know that for a given circle of radius r, the Range: (−∞, c − a ] ∪ [c + a, ∞ )
radian measure θ inside the circle (central angle) is 2π
Period:
related to the arc length s subtended by θ . Then s, b
π
which is a linear unit is numerically equal to θ which Vertical Asymptotes: x = d + (2n + 1)
2b
is a radian measure. Thus the term “circular
Phase Shift: d to the left if d < 0 , d to the right if
function” denotes the function wherein a
d >0.
trigonometric function of angle θ which were
f. COSECANT : y = c + a csc b(x − d )
found by choosing a point (x,y) of the circle can be
where:
written as a function of its subtended arc s, a real
 nπ 
number. Domain :  x x ≠ d + , n ∈ Ζ
 b 
Range: (−∞, c − a ] ∪ [c + a, ∞ )
a. SINE : y = c + a sin b(x − d )
where: 2π
Period:
Domain : (−∞, ∞ ) b
Range: [c − a, c + a ] Vertical Asymptotes: x = d +

for n ∈ Ζ
2π b
Period: Phase Shift: d to the left if d < 0 , d to the right if
b
Amplitude: a d >0.
Phase Shift: d to the left if d < 0 , d to the right if
d >0. EXAMPLE:
b. COSINE : y = c + a cos b(x − d )
where:  Find the vertical asymptotes of the graph of the
Domain : (−∞, ∞ )  π
curve y = −2 + 3 sec 2 x +  .
Range: [c − a, c + a ]  4

Period:
b

35
Solution:
 π  π
y = −2 + 3 sec 2 x +  = −2 + 3 sec 2 x + 
 4  8
Thus, the vertical asymptotes are:
π
x=d+ (2n + 1) = − π +
π
(2n + 1)
2b 8 2(2)
π nπ
x= + ; n∈Ζ
8 2

EXPANSION OF tan(nx)

In this procedure, we will use a modified Pascal’s


Triangle to easily determine the expansion of tan(nx) in
terms of powers of tan x . As you can see, every two
entries in each row alternates sign.

1
1 1
1 2 −1
1 3 − 3 −1
1 4 −6 −4 1
1 5 − 10 − 10 5 1
M

To express tan(nx) in terms of tan x , look at the (n + 1)th


row and apply the formula:

n tan x − n C3 tan 3 x + n C5 tan 5 x − K


tan( nx) =
1−n C2 tan 2 x + n C4 tan 4 x − K

EXAMPLE:

 Express tan (5 x ) in terms of tan x .

Solution:

Looking at the 6th row of the Pascal’s Triangle, the


coefficients on the numerator part of the formula
above is just the even-placed entries in that row
while the coefficients on the denominator is just the
odd-placed entries on that same row of the
Pascal’s Triangle. Thus:

5 tan x − 10 tan 3 x + tan 5 x


tan 5 x =
1 − 10 tan 2 x + 5 tan 4 x

36
TEST 9: MISCELLANEOUS QUESTIONS:

1. How many odd entries are there in the 1. Solve for all prime number solutions a, b and
143rd row of the Pascal’s Triangle? c to the equation:

2. How many even entries are there in the a3 = c + b3


14344th row of the Pascal’s Triangle?
2. If
3. Express the 11th
square number into sum of
two consecutive triangular numbers. x = 3 sin θ − sin 3θ ; y = cos 3θ + 3 cos θ ,

4. Express the 25th hexagonal number into 2 2


sum of two consecutive triangular Evaluate x 3 + y 3 .
numbers.
x 2 − 2x + 1
5. The sum of the 21st
square number and 22nd 3. If f (x ) = , then f −1
(1) is what?
x 2 − 4x + 7
square number is equal to the nth
pentagonal number. Find n.
4. Find the smallest positive number that is
divisible by 13 and leaves a remainder of 1
6. Find the sum of all the digits of all the
when each divided by 2, 3, 4, … , 12.
integers from 1 to 143.

5. Find all two-digit integers that is twice the


7. Find the sum of all the digits of all the
sum of its digits.
integers from 1000 to 10000.

8. If S (n ) denotes the sum of the digits of n,


then what is the exact value of:

S (2 ) + S (4 ) + S (6 ) + S (8) + K + S (2008)

9. Find the domain and range of the function:

x 
y = −5 + 4 csc − π 
4 

10. Find the vertical asymptotes of the graph


of the function:

 x 5π 
y = 2008 − 2009 cot + 
2 3 

11. Express tan 7 x in terms of powers of tan x .

12. Express tan 15 x in terms of powers of tan x .

37
LAPLACE’S METHOD OF COFACTORS 3

AND MINORS det( A) = ∑ C 2 j = C 21 + C 22 + C 23


j =1

23 30 11 30
Laplace’s Method of Cofactors and Minors – is a det( A) = ( −1) 2 +1 ⋅13 + (−1) 2+ 2 ⋅ 0
special method for obtaining the determinant of 7 10 − 1 10
any square matrix. 11 23
+ (−1) 2+3 ⋅ 4
−1 7
Suppose that for a matrix of size n = 4 , we choose a
det( A) = −13( 23 ⋅ 10 − 7 ⋅ 30) + 0 − 4(11 ⋅ 7 + 1 ⋅ 23)
specific row (i ) or column ( j ) , say i = 4:
det( A) = −13( 20) − 4(100) = −660

 a11 a12 a13 a14 


GAUSS-JORDAN METHOD
 
a 21 a 22 a 23 a 24 
 a 31 a 32 a 33 a 34  Gauss-Jordan Method – is a method for solving the
 
a 41 a 42 a 43 a 44  values of the unknowns in simultaneous linear
equations by the use of matrix-elementary row
Starting from a 41 , all elements in the chosen row will operations. This method was named after
mathematicians Carl Friedrich Gauss and Camille
produce its minor denoted by M ij and cofactor
Jordan.
denoted by C ij . So for a 41 :
PROCESS:
a12 a13 a14
STEP 1: For the simultaneous linear equations (where
MINOR : M 41 = a 22 a 23 a 24
the number of unknowns equal the number of
a 32 a 33 a 34 equations),

a11 x1 + a12 x 2 + K + a1n x n = b1


COFACTOR : C 41 = (−1) i + j a 41 ⋅ M 41 = −a 41 ⋅ M 41
a 21 x1 + a 22 x 2 + K + a 2 n x n = b2
M M M
Therefore, we can summarize the method as:
a n1 x1 + a n 2 x 2 + K + a nn x n = bn

LAPLACE’S METHOD OF COFACTORS AND MINORS Obtain a corresponding matrix of the form:

To find the determinant of matrix A,


 a11 a12 L a1n b1 
 
STEP 1: From the square matrix A of size n, choose a a 21 a 22 L a 2n b2 
specific row or column, say row 1 (i = 1) .  M M O M M
 
 a n1 an2 L a nn bn 
 a11 a12 a13 L a1 j 
 
a 21 a 22 a 23 L a2 j  STEP 2: Transform the matrix from the left of the
 M M M O M  augmenting line to an identity matrix by
  performing series of matrix elementary row
 a i1 ai2 a i3 L a ij 
operations, that is:

STEP 2: Now, the determinant of A is obtained by 1 0 L 0 x1 


getting the sum of all the cofactors:  
0 1 L 0 x2 
n n M M O M M 
 
det( A) = A = ∑ Cij = ∑ C ij 0 0 L 1 x n 
j =1 i =1

Note: As you can see, the values from the right of the
EXAMPLE: augmenting line will serve as the solutions to the
11 23 30 problem.
 
 Find the determinant of matrix A: A = 13 0 4  .
−1 7 10 EXAMPLE:

Solution:
 Find the values of x, y and z from the simultaneous
Choosing row 2 of matrix A:
linear equations:

38
x+ y+z =8 2 a − b + c − 2d = 6
2 x + 3 y − 5 z = −1 a + 4b + 3c − d = 12
3x − 2 y + 4 z = 7 − a + 3b − 2c + d = 10

Solution:
The simultaneous linear equations above are AMERICAN INVITATIONAL MATHEMATICS EXAM
equivalent to an augmented matrix: QUESTIONS:
1 1 1 8
  4. AIME 1983, Question No. 2
 2 3 − 5 − 1
3 − 2 4 7 
  Let f ( x) = x − p + x − 15 + x − p − 15 , where p ≤ x ≤ 15 .
Performing matrix elementary row operations:
Determine the minimum value taken by f (x ) by x
1 1 1 8 1 1 1 8
    in the interval 0 < p < 15 .
2 3 − 5 − 1 2 R1 − R2 = 0 − 1 7 17 
3 − 2 4 7  3R1 − R3 0 5 − 1 17 
   
5. AIME 1983, Question No. 6
1 1 1 8 1 1 1 8 
   
 0 − 1 7 17  − R2 = 0 1 − 7 − 17  Let an equal to 6n + 8n . Determine the remainder
0 5 − 1 17  0 5 − 1 17 
    upon dividing a83 by 49.
1 1 1 8  R1 − R2 1 0 8 25 
   
 0 1 − 7 − 17  =  0 1 − 7 − 17  6. AIME 1984, Question No. 2
0 5 − 1 17  5 R2 − R3 0 0 − 34 − 102
   
The integer n is the smallest positive multiple of 15
1 0 8 25  1 0 8 25 
    such that every digit of n is either 8 or 0. Compute
0 1 − 7 − 17  = 0 1 − 7 − 17
n
0 0 − 34 − 102 1  3  .
 − R3 0 0 1 15
34
1 0 8 25  R1 − 8R3 1 0 0 1  → x
    7. AIME 1984, Question No. 4
0 1 − 7 − 17  R 2 +7 R3 = 0 1 0 4 → y
0 0 1 3  0 0 1 3 → z
   Let S be a list of positive integers (not necessarily
Therefore, distinct) in which the number 68 appears. The
x =1; y = 4 ;z = 3 arithmetic mean of the numbers in S is 56.
However, if 68 is removed, the arithmetic mean of
the numbers is 55. What’s the largest number that
TEST 10: can appear in S?

1. Find the determinant of the matrix below: 8. AIME 1985, Question No. 4

1 2 3 A small square is constructed is constructed inside


  a square of area 1 by dividing each side of the
A = − 5 0 − 4
 − 1 4 3  unit square into n equal parts, and then
connecting the vertices to the division points
closest to the opposite vertices. Find the value of
2. What is the determinant of: 1
n if the area of the small square is exactly
1985 .
x 2 −1 5 
 
0 9 0 − 2
A(x ) = 
x 0 − 2 x a34 
 
6 1 − 3 − 7 

3. Find the values of a, b, c and d given the


following system of linear equations:

a+b+c+d =8

39
9. AIME 1985, Question No. 7
(52 + 6 43 ) − (52 − 6 43 )
3 3
2 2

Assume that a, b, c and d are positive integers


such that a 5 = b 4 , c3 = d 2 and c − a = 19 .
Determine d − b .

10. AIME 1986, Question No. 3

If tan x + tan y = 25 and cot x + cot y = 30 , what is


tan (x + y ) .

11. AIME 1986, Question No. 7

The increasing sequence:


1, 3. 4, 9, 10, 12, 13,K
consists of all those positive integers which are
powers of 3 or sums of distinct powers of 3. Find
the 100th term of this sequence.

12. AIME 1987, Question No. 2

What is the largest possible distance between two


points on the sphere of radius 19 with center (-2,-
10,5) and the other on the sphere of radius 87 with
center (12,8,-16)?

13. AIME 1987, Question No. 14

Compute:
(10 + 324)(22 + 324)(34 + 324)(46 + 324)(58 + 324)
4 4 4 4 4

(4 + 324)(16 + 324)(28 + 324)(40 + 324)(52 + 324)


4 4 4 4 4

14. AIME 1988, Question No. 9

Find the smallest positive integer whose cube


ends in 888.

15. AIME 1989, Question No. 5

When a certain biased coin is flipped five times,


the probability of getting heads exactly once is
not equal to 0 and is the same as that of getting
i
heads exactly twice. Let , in lowest terms, be
j
the probability that the coin comes up heads in
exactly 3 out of 5 flips. Find i + j .

16. AIME 1990, Question No. 2

Evaluate:

40
TEST 11

1
1. If sin x + cos x = , then what is the value of
2
sin 3 x + cos3 x ?
2. Suppose that AB = AC = CD and AD = BD . What is 11. A bowl contains 100 pieces of colored candy: 28
the measure of ∠ADC in degrees? green, 20 red, 12 yellow, 10 blue, 20 brown and 10
orange. If you are blindfolded as you pick and
eat candy from this bowl, then how many pieces
must you eat in order to guarantee that you have
eaten at least 15 of the same color?
12. Suppose that F (x ) = f (g (x )) and g (3) = 6 , g ′(3) = 4 ,
3. At a party, every two people shook hands once. f ′(3) = 2 and f ′(6 ) = 7 . Find F ′(3) .
How many people attended the party if there 13. Michael and Dave play a game in which each
were 66 handshakes? independently throws a dart at a target. Michael
4. Find the numbers a and b such that hits the target with probability of 0.6, while Dave
ax + b − 2 hits the target with probability of 0.3. Michael wins
lim =1
x→0 x the game if he hits the target and Dave misses.
5. Dave can answer each problem on a certain test Dave wins if he hits the target and Michael misses.
in 6 minutes. Michael can answer each problem Otherwise the game is a tie. What is the
in 1 minute. Suppose Michael rests for two hours in probability that the game is a tie?
the middle of answering the problems but Dave 14. Evaluate:
works straight through the test without stopping. 1 1 1 1
+ + +K+
Suppose further that they finish the test at the log 2 (100!) log3 (100!) log 4 (100!) log100 (100!)
same time. How long did it take Dave to answer 15. The perimeter of a right triangle is 12 + 8 3 . The
all the problems?
sum of the squares of all three of its sides is 294.
6. Suppose that
Find the area of this triangle.
f ( x) = x5 + ax 4 + bx3 + cx 2 + dx + e 16. The integer n is obtained by reversing the order of
and that f (1) = f ( 2) = f (3) = f ( 4) = f (5) . Then what the digits of the 3-digit integer m. If the product of
is the value of a? n and m is equal to 214875, then what is the
7. Let two 8 x 12 rectangles share a common corner middle digit of n?
and overlap the distance from the bottom right 17. The product

(x + 5)(x + 10)(2 x 2 + 3)(x3 + 6 x + 16) (x + 9)(x + 4)3 (x + 18)


corner of one rectangle to the intersection point 2

along the right edge of that rectangle is 7. What is


is expanded. In the resulting polynomial, how
the area of the shaded region?
many of the coefficients are odd?
18. If f (x ) satisfies 2 f (x ) + f (1 − x ) = x 2 for all x, then
what is f (x ) ?
19. When written in base 8, the number n is
34112d4357 where d denotes a digit in base 8. If n
is divisible by 7, then what is the value of d?
20. If

f (x ) = 2
8. Find the 46th digit after the decimal in the decimal 1
1 x +x
expansion of .
1996 Find f 10 (x ) where f 10 (x ) means 10th derivative of
9. Find the horizontal and vertical asymptotes of the f (x ) .
graph of the function
21. If (3x + 5)100 is expanded, what is the largest power
2x + 1 2
f (x ) = of 2 that divides the coefficient of x39 ?
3x − 5
22. The interior of a square contains 30 points. This
10. Suppose that AB = AC = 4 , ∠CAB is a right angle
square, along with its interior, is partitioned into
and P is the midpoint of AB. What is the smallest
non-overlapping triangles, so that the vertices of
possible value of the perimeter of ∆PMN ?
the triangle consist of these 30 interior points

41
together with the four corners of the square. If no to R is 2 inches). What is the length of the shortest
three of these 34 points are collinear, then how path in inches that the bug could have made
many triangles can you form? from P to R?
23. The Arithmetic-Geometric Mean Inequality asserts
a+b
that if a ≥ 0 and b ≥ 0 , then ≥ ab and
2
equality holds if and only if a = b . Given that x > y > 0
and xy = 2 , what is the smallest possible value of
x2 + y 2
?
x− y

33. For how many primes p is the value of p 2 + 21 p − 1


24. Evaluate:
 π  2π  3π  4π  also a prime?
 sin  sin  sin  sin  34. Suppose we draw 100 horizontal lines and 100
 5  5  5  5 
vertical lines in the plane, how many “pieces” of
25. How many integers x in {1, 2, 3, K , 99, 100} are there
the plane are formed by cutting along all of these
such that x 2 + x 3 is the square of an integer? lines? (Note: Some of the pieces will have infinite
26. Suppose that for some numbers a, b and k, we area)
have the factorization 35. Evaluate
( )(
x 4 + 2 x3 + kx 2 − x + 2 = x 2 + ax + 1 x 2 + bx + 2 ) sin 2 15o cos 2 15o
What is the value of k? 36. How many points do the graphs of 4 x 2 − 9 y 2 = 36
27. The coordinates for A and D are (7,4 ) and (−5,−3)
and x 2 − 2 x + y 2 = 15 have in common?
respectively. What is the shortest possible length of 37. Define a sequence by a1 = 1 and for n ≥ 1
a path ABCD where B is a point on the line y = 2 ;
0 if an = 0 and n is odd
C is a point on the line y = 0 , and the line 
2 if an = 0 and n is even
segment BC is perpendicular to the line y = 0 ?
an +1 = 1 if an = 1 and n is odd
0 if an = 1 and n is even

1 if an = 2
How many of the numbers a1, a2 , a3 ,K, a100 are
equal to 2?
28. How many 9’s are there in the decimal expansion 38. What is the graph of the equation

of (99999899999 )2 ? x 2 − xy + x − y = 0
29. Find a third-degree polynomial Q such that 39. What is the coefficient of x18 in the polynomial
Q (1) = 1 , Q′(1) = 3 , Q′′(1) = 6 and Q′′′(1) = 12 . (1 + x )20 + x(1 + x )19 + x 2 (1 + x )18 + K + x18 (1 + x )2
30. Ten squares of equal size are arranged in the grid 40. Solve for the first derivative of f (x ) given that
below. What is the value of β − α ?
f (x ) = x x x x
41. There are four cowboys in a saloon. At midnight,
each cowboy randomly chooses one of the other
three cowboys and shoots him. What is the
probability that exactly two cowboys are shot?
31. What is the remainder when 42. The following inequalities hold for all positive
x 200 − 2 x99 + x50 − 2 x 49 + x 2 + x + 1 integers n:
is divided by (x − 1)(x − 2 ) . n +1 − n <
1
< n − n −1
32. A cylindrical can is six inches tall and its base is 4n + 1
four inches in diameter. A bug crawls from a point What is the greatest integer which is less than
24
P on the upper rim of the can once around the
can to a point Q which is four inches directly ∑
n =1
1
4n + 1
below P. Then the bug crawls from Q once
around the can to a point R on the bottom rim of
the can directly below P (so the distance from Q

42
43. Consider the points A(−5,−1) , B(− 1,0) , C (1,2 ) and
D (1,3) . Let P be a point and let
d = PA2 + PB 2 + PC 2 + PD2
so that d is the sum of the squares of the distances
from P to each of A, B, C and D. What is the least
possible value for d?
44. Let a, b and c be the three roots of x3 − 64 x − 14 .
What is the value of a 3 + b3 + c3 ?
45. Simplify:
3
2+ 5 +3 2− 5
46. Let ABCD be a rectangle and let P be a point
inside the rectangle. If PA = 8 , PB = 4 and PD = 7 ,
then what is the length of PC?

47. Find the domain of the function

f (x ) = 1 − 2 − 3 − x
48. Find
 n k 
1
lim 
n →∞  n


k =1

en 


49. Determine the tens digit of the sum
0!+1!+2!+3!+ K + 9999!+10000!
50. If
F ( x ) = 5 x 5 − 4 x 4 + 7 x 3 + 8 x 2 − x − 1 = G −1 ( x )
Find G (F (F (G (x )))) .

43
WORK PROBLEMS  How much tin and how much lead must be added
to 700 kg. of an alloy containing 50% tin and 25%
Principles: lead to make an alloy which is 60% tin and 20%
lead?
A : rate of wor ker A
Solution:
t A : time A worked
A(t A ) : amount of work done by A for time t Let x be the amount of tin and y be the amount of
lead to be added on the original alloy.
Key Relationships:
Considering tin content:
For a completed work, the amount of work done is
equal to 1. Thus, 0.5(700) + x + 0 y = 0.6(700 + x + y )
0.4 x − 0.6 y = 70 → eqn.1
A(t A ) = 1 → A =
1 1
or t A =
tA A Considering lead content:

EXAMPLE: 0.25(700) + 0 x + y = 0.2(700 + x + y )


0.2 x − 0.8 y = 35 → eqn.2
 Ding can finish a job in 8 hours. Tito can do it in 5
hours. If Ding worked for 3 hours and then Tito was
Solving for equations 1 and 2, we have:
asked to help him finish it, how long Tito will have
to work with Ding?
x = 175 kg. ; y = 0 kg
Solution:
Thus, add 175 kg. of tin and no lead.
Let t be the time Tito worked with Ding to finish the
job. Therefore: SUM OF AGES PROBLEMS

1 1 1 1 Principle:
D= = and T = =
tD 8 tT 5
Represent the average age of group of people whose
number is unknown.
And from the problem:

D(3) + (D + T )(t ) = 1
sum of ages
average = → sum of ages = n(average)
n
1
(3) +  1 + 1 (t ) = 1
8 5 8 EXAMPLE:
25
t= hrs.  The sum of parents’ ages is twice the sum of their
13
children’s ages. Five years ago, the sum of the
parents’ ages is four times the sum of their
MIXTURE PROBLEMS
children’s ages. In 15 years, the sum of the
parents’ ages will be equal to the sum of their
Principle:
children’s ages. How many children are there?

c : percent concentration of solute


Solution:
A : amount of mixture
cA : amount of solute or any fractional part Let:
n : number of children
A : average age of the children at present
A – 5 : average age of the children 5 years ago
A + 5 : average age of the children in 15 years
EXAMPLE:

Consider the sum of ages of parents and children:

44
(100 x + 10 y + z ) − 396 = 100 z + 10 y + x
was is will be x − z = 4 → eqn.3
Parents 4 n( A − 5 ) 2nA n( A + 15)
Children n( A − 5) nA n( A + 15) Substitute the value of x from equation 2 to equation 3
we have:
Consider the time elapsed from past to present for
parents: z=4

2nA − 4n( A − 5) = 10 And it follows that x = 8 and y = 17 − x − z = 5 . Thus


− nA + 10n = 5 → eqn.1 the number we are looking for is 854.

Consider the time elapsed from present to future for CLOCK PROBLEMS
parents:
Principle:
n( A + 15) − 2nA = 30
Let:
− nA + 15n = 30 → eqn.2
H : distance traveled by the hour hand
M : distance traveled by the minute hand
Solving equations 1 and 2 we have:
S : distance traveled by the second hand

n=5 CASE 1: If the minute and hour hand were involved,


you may use the formula:
Therefore, there are 5 children in the family.

NUMBER-DIGIT PROBLEM M =
2
(∠reference ± ∠required )
11
Principle:
Please take note the following:
 Use positive sign when minute hand is ahead of
If there is a three-digit number for example:
the hour hand.
 Use negative sign when minute hand is behind the
Let:
hour hand.
x : hundred’s digit
 ∠reference : angle between the hands of the
y : ten’s digit
z : unit’s digit clock during preceding exact time.
 M in the formula will give us an answer in unit of
the number = 100 x + 10 y + z minutes.

the number with reversed digits = 100 z + 10 y + x


CASE 2: Otherwise, use the following formulas:

EXAMPLE: M
H= ; S = 60M
12
 The sum of the digits of a three-digit number is 17.
The hundred’s digit is twice the unit’s digit Subtract
EXAMPLE:
396 from the number and the order of the digits
will be reversed. Find the number.
 At what time after 3’oclock will the hands of the
clock be bisected by the second hand?
Solution:

Solution:
If you will draw the analog clock, we can see that:

From the problem:


M
x + y + z = 17 → eqn.1 15 + = M + 2θ → eqn.1
12
x = 2 z → eqn.2
But θ = 60M − M = 59 M , thus:

45
Find the exact time when he left and the exact
= M + 2(59M )
M
15 + time when he returned.
12
10
180  1 
M = min . 11. In the expansion of  x 2 −  , find the constant
1427  x3 
 180  10800 term (i.e. term free of x).
S = 60 M = 60 = sec s
 1427  1427 12. Reversing the digits of Ana’s age gives her
mother’s age with a difference of 18 years. If the
sum of the digits of each age is 6, how old is Ana?
10800
So the required time is 3 : 00 : . 13. Mike, Louie and Joy can mow the lawn in 4, 6 and
1427 7 hours respectively. What fraction of the yard
can they mow in 1 hour if they work together?
TEST 12 14. The sum of the digits of a two-digit number is 11. If
the digits are reversed, the resulting number is
1. Pedro can paint a fence 50% faster than Juan seven more than twice the original number. What
and 20% faster than Pilar and together they can is the original number?
paint a given fence in 4 hours. How long will it 15. A father is now 41 and his son 9. After how many
take Pedro to paint the same fence if he had to years will his age be just triple his son’s age?
work alone? 16. The time required for two examinees to solve the
2. Equal volumes of two different liquids evaporate same problem differs by two minutes. Together
at different, but constant rates. If the first is totally they can solve 32 problems in one hour. How long
evaporated in 6 weeks, and the second in 7 will it take the slower problem solver to solve a
weeks, when will be the second be ½ the volume single problem?
of the first? 17. Pipes A and B can fill an empty tank in 6 and 3
3. For a particular experiment, you need 5 liters of a hours respectively. Drain C can empty a full tank
10% solution. You find 7% and 12% solution on the in 24 hours. How long will an empty tank be filled if
shelves. How much of the 7% solution should you pipes A and B run with drain C open?
mix with the appropriate amount of the 12% 18. The ten’s digit of a two-digit number is 1 less than
solution to get 5 liters of a 10% solution? twice the unit’s digit. They differ by 4. Find the
4. If y varies directly as x2 and y = 9 when x = 2 , number.
what is the value of x when y is 1? 19. How many minutes after 4’oclock will the hands of
5. The arithmetic mean of six numbers is 17. If two the clock form an angle of 120 degrees for the first
numbers are added to the progression, the new time?
set of numbers will have an arithmetic mean of 19. 20. Find the term involving x 3 yz 2 in the expression

( x + 2 y + 3 z )6 .
What are the two numbers if their difference is 4?
6. A and B can do a piece of work in 20 days, B and
C in 30 days, C and A in 40 days. If the three work 21. Find the term involving a 3 b 5 c 2 d in the expansion
of (2a − b + 3c + 2d ) .
together, how long will they be able to finish the 11
job?
7. A pipe can fill up the tank with the drain open in 3 22. My daughter is twice as old as my son and half as
hours. If the pipe runs with the drain open for one old as I am. In 22 years, my son will be half my
hour and then the drain is closed, it will take 45 age. How old is my daughter?
more minutes for the pipe to fill up the tank. If the 23. When its digit are reversed, a particular positive
drain will be closed right at the start of filling, how two-digit integer is increased by 20%. What is the
long will the pipe be able to fill up the tank? original number?
8. The sum of the digits of a 3-digit number is 17. The 24. Reid is twice as old years as Gabe. Four years
hundred’s digit is twice the unit’s digit. Find the ago, Gabe was twice as old as Dani. In 10 years,
number. Reid will be twice as old as Dani. How many years
9. At what time after 5’oclock will the minute and old is Reid now?
hour hands of an analog clock be opposite with 25. In a three-digit number, the sum of the hundred’s
each other? digit and ten’s digit is 2 more than twice the units
10. A liaison officer left the office at past 2’oclock. digit. The sum of the ten’s digit and the unit’s digit
After 2 to 3 hours (upon returning), he noticed that is equal to the hundred’s digit. The sum of the
the hands of the clock were reversed (i.e. three digits is 14. Find the number.
interchanged in position as to that when he left).

46
UNIVERSITY OF SOUTH CAROLINA 11. Simplify:
9+4 2
1. If 4 x − 4 x −1 = 24 , then what is the exact value of 12. What is the sum of the measures of the angles A,
(2 x )x ? B, C, D and E in the figure below?
2. A bus travel up a hill at an average speed of 50
kph. At what average speed would it have to
travel down the hill to average 60 kph for the
entire trip?
3. What is the area of the largest triangle that can
be inscribed in a semicircle of radius r?
4. Find the exact value of 13. I have two dice, one red and one blue. When the
 7π  two dice are rolled, what is the probability that
sec  
 12  the number showing on the red die is larger than
5. A cube measuring 100 units on each side is the number showing on the blue die?
painted only on the outside and cut into unit 14. For a certain integer n, 5n + 16 and 8n + 29 have a
cubes. How many cubes are there with paint only common factor lager than one. Find that
on two sides? common factor.
6. For what values of a does the system of 15. A certain function f satisfies
equations: f (x ) + 2 f (6 − x ) = x

x2 = y2 for all real numbers x. Find the value of f (1) .


16. If the line y = 2 x + b is tangent to the circle
(x − a )2 + y 2 = 1
have exactly three solutions? x + y = 1 , then what are all possible value(s) of
2 2

7. Reading from left to right, what is the 8th digit of b?


the product 17. Evaluate:
7216848248168566432 x 125 tan(1o ) tan(2o ) tan(3o ) ⋅ K⋅ tan(88o ) tan(89o )
8. Given a square whose sides have length 2a, find 18. There are 120 five-digit numbers that can be
the area of the region bounded by the four formed by permuting 1, 2, 3, 4 and 5 such as
semicircles which are in the interior of the square 12345, 12354, 21435, K , 54321
and have the four sides of the square as find the sum of all these numbers.
diameters. 19. Two parallel chords in a circle have lengths 6 and
8. The distance between them is 1. Find the
diameter of the circle.

9. In the configuration below consisting of 36 one-


by-one squares, how many total squares with
horizontal and vertical sides can be formed using 20. In base 10 arithmetic, what value of the digit d for
the points as vertices? which the number d456d is divisible by 18?
21. Find the remainder when x100 − 2 x99 + 4 is divided
by x 2 − 3 x + 2 .
22. What is the sum of the squares of the roots of
x3 + 2 x 2 − 2 x − 2 ?
23. If x satisfies the equation
10. Let lu be the normal line to y = x 2
( ) . Let
at u,u 2
tan x =
sin 10o + sin 40o
(0, bu ) be the y-intercept of lu . Then what is the cos10o + cos 40o
value of: and x is between 0 and 90 degrees, then x is
lim bu equal to what?
u→0 24. Points A and B are in the 1st quadrant and O is the
origin. If the slope of OA is 1, the slope of OB is 7,

47
and the length of OA is equal to the length of (3x 8
)
− 2 x 6 + x5 + 2 x 4 − x 2 + 1 = a0 + a1x + a2 x 2 + K + a40 x 40
OB , determine the slope of AB .
25. The integers 2, 3, 4, … are arranged in columns as determine the sum a0 + a2 + a4 + K + a40 .
shown below 37. What is the radius of the circle which
circumscribes a triangle with sides of lengths 2, 3
and 4?
38. Evaluate:
cos 20o cos 40o cos 80o
39. Circle C is inscribed in square PQRS, and the
length of segment PQ is 1. Inside square PQRS,
in what column does 1000 fall? circle D is tangent to circle C, segment PQ and
26. In the right triangle ABC below, AD = AE and segment QR. Find the radius of circle D.

CF = CE . If angle DEF equals xo , find x.

40. Evaluate:
6
 x2 3 
99
27. In the binomial expansion of  −
 3 x2 
 
, what is
∑ log k k+ 1 
k =1
the value of the constant term? 41. How many integers x are there such that
28. Evaluate:
1 ≤ x ≤ 100 and x3 + 4 x + 2 is divisible by 7?
12 − 2 2 + 32 − 42 + K − 19982 + 19992
42. What is the largest integer k such that 5k divides
29. Suppose that x and y are positive integers, c is a
3(10!) + 12(5!) + 4(7!)
real number, and i is the imaginary number. If
43. Evaluate:
(x + iy )3 = −74 + ci , what is the only possible value of
1 1 1 1
the ordered pair (x,y)? + + +K+
1⋅ 2 2 ⋅ 3 3 ⋅ 4 99 ⋅ 100
30. If x < −2 , simplify:
44. If three circles of radius 1 are mutually tangent as
1− 1+ x shown, what is the area of the gap they enclose?
1
31. What is the binary representation of ?
5
32. An 8-inch chord is twice as far from the center of
a circle as a 10-inch chord. Find the
circumference of the circle.
33. If x and y are real numbers and x 2 + y 2 = 1 , then
what is the maximum value of (x + y )2 . 45. If (1 + i )100 is expanded and written in the form
34. An urn contains N black marbles and N white a + bi where a and b are real numbers, then what
marbles. Three marbles are chosen from the urn
is the value of a?
randomly and without replacement. What is the
46. Find the real roots of the quartic:
value of N if the probability that all three chosen
x 4 + 7 x 3 + 13x 2 + 28 x + 36 = 0
1
are white is ?
2
35. A sequence {ai } is defined as follows:
47. Evaluate:
100

ai +1 =
1
1 − ai
for i ≥ 1 ∏1 − n1 
n =2
2

if a3 = a1 , then what is the value of (a9 )9 ? 48. Two circles, one of radius 8 and one of radius 18 is
tangent. There are two lines each of which is
36. In the expansion
tangent to both circles, as shown in the diagram.

48
Find the distance from the intersection of these ( )
10 cot cot −1 3 + cot −1 7 + cot −1 13 + cot −1 21
lines to the center of the circle with radius 8.

6. 1985, Question No. 2

When a right triangle is rotated about one leg, the


volume of the cone produced is 800π cm3 . When
the triangle is rotated about the other leg, the
49. In the figure below, the lines AN, AM and BC are volume of the cone produced is 1920π cm3 . What
tangent to the circle, and the length of AN is 7. is the length (in cm.) of the hypotenuse of the
What is the perimeter of triangle ABC? triangle?

7. 1985, Question No. 13

The numbers in the sequence 101, 104, 109, 116,


… are of the form an = 100 + n 2 , where n = 1,2,3,K
For each n, let d n be the greatest common
50. Find the smaller angle formed between the
minute and hour hands of an analog clock at 2:15 divisor of an and an +1 . Find the maximum value of
pm. d n as n ranges through the positive integers.

AMERICAN INVITATIONAL MATHEMATICS EXAM 8. 1986, Question No. 1

1. 1983, Question No. 1


What is the sum of the solutions to the equation:
12
Let x, y and z all exceed 1, and let w be a positive
4
x =
7−4 x
number such that log x w = 24, log y w = 40 , and
log xyz w = 12 . Find log z w .
9. 1986, Question No. 5

2. 1983, Question No. 3 What is the largest positive integer n for which
n3 + 100 is divisible by n + 10 ?
What is the product of the real roots of the
equation
10. 1986, Question No. 11

x 2 + 18 x + 30 = 2 x 2 + 18 x + 45
The polynomial 1 − x + x 2 − x3 + K + x16 − x17 may be
written in the form a0 + a1 y + a2 y 2 + K + a16 y16 + a17 y17 ,
3. 1983, Question No. 9
where y = x + 1 and that ai ' s are constants. Find

Find the minimum value of the value of a2 .

9 x sin x + 4
2 2

x sin x
for 0 < x < π . 11. 1987, Question No. 4

4. 1984, Question No. 5 Find the area of the region enclosed by the graph
x
of x − 60 + y = .
4
Determine the value of ab if log8 a + log 4 b 2 = 5 and

log8 b + log 4 a 2 = 7 . 12. 1987, Question No. 8

5. 1984, Question No. 13 What is the largest positive integer n for which
there is a unique integer k such that
Find the value of

49
8 n 7 1 1 2
< < + − =0
15 n + k 13 x − 10 x − 29
2
x − 10 x − 45
2
x − 10 x − 69
2

13. 1987, Question No. 9 20. 1990, Question No. 5

Triangle ABC has right angle at B, and contains a Let n be the smallest positive integer that is a
point P for which PA= 10, PB = 6, ∠APB= ∠BPC= ∠CPA. multiple of 75 and has exactly 75 positive integral
Find PC. n
divisors, including 1 and itself. Find .
75
14. 1988, Question No. 5

m
Let , in lowest terms, be the probability that a
n
randomly chosen positive divisor of 1099 is an
integer multiple of 1088 . Find m + n .

15. 1988, Question No. 8

The function f, defined on the set of ordered pairs


of positive integers, satisfies the following
properties:
f ( x, x ) = x
f ( x, y ) = f ( y , x )
( x + y ) f ( x, y ) = y f ( x , x + y )

Calculate f (14,52)

16. 1989, Question No. 1

Compute:
31 ⋅ 30 ⋅ 29 ⋅ 28 + 1

17. 1989, Question No. 4

If a < b < c < d < e are consecutive positive integers


such that b+c+d is a perfect square and
a + b + c + d + e is a perfect cube, what is the
smallest possible value of c?

18. 1990, Question No. 3

Let P1 be a regular r-gon and P2 be a regular s-


gon (r ≥ s ≥ 3) such that each interior angle of P1
58
is as large as each interior angle of P2 . What is
59
the largest possible value of s?

19. 1990, Question No. 4

Find the positive solution to

50

You might also like